Download as pdf or txt
Download as pdf or txt
You are on page 1of 135

‫‪Mohammad‬‬

‫‪Sami‬‬
‫ﺗﺠﻤﯿﻌـــــــــــــــــــــــــﺎت ‪0202‬‬
‫‪p‬‬
‫ا ﯾﺼﻞ اﻟﻨﺎس اﻟﻰ ﺣﺪﯾﻘﺔ اﻟﻨﺠﺎح ‪ ،‬دون ان ﯾﻤﺰوا ﺑﻤﺤﻄﺎت اﻟﺘﻌﺐ و اﻟﻔﺸﻞ و‬
‫اﻟﯿﺄس ‪،‬و ﺻﺎﺣﺐ اإﻟﺮادة اﻟﻘﻮﯾﺔ ال ﯾﻄﯿﻞ اﻟﻮﻗﻮف ﻓﻲ ھﺬه اﻟﻤﺤﻄﺎت‬

‫‪Never give up. Great things take time. Be‬‬


‫‪patient‬‬

‫أ‪ /‬ﻣﺣﻣد ﺳﺎﻣﻲ ‪// 2996699650‬‬










Medication and medication calculation

1. Diabetic Patient with history of hypertension hyperthyroidism hyperlipidemia is going for a


surgery. Which medication can be hold while the patient is NPO‫ﻻﻧﮭﺎ ﺗﻧزل اﻟﺳﻛر‬
A. Lispro
B. Atorvastatin
C. Glucophage ✅
D. Thyroxine

2. Soldier was brought to triage area after being exposed to chemical weapons. Signs and
symptoms of nerve gas exposure were noticed. A nurse prepares for medical
management. Which medication should the nurse prepare for the patient?
A. Atropine ✅
B. Adrenaline
C. Sodium nitrate
D. Sodiumthiosulphate

3.Patient preoperative take midazolam hydrochloride. The patient become restless and suffered
from respiratory distress and confusion. What should the nurse and surgical team do?
A. Defibrillate patient
B. Administration antidote (activated characol) ✅
C. Administration epinephrine
D. Ventilationpatientwithoxygentherapy

4.What is the side effects of isoniazid?


Urecemia
Photosensitivity ✅ ‫ﺣﺳﺎﺳﮫ ﻟﺿوء‬
Nerve inflamation

5.Girl come hospital with thalassemia . She is given blood transfusion every 3 weeks. She came
to hospital for blood transfusion and the doctor ordered lab investigation before administeration,
He found on results increase ferritin level. What should the nurse give for her to prevent
complication?
A. deferasirox ✅
B. Iron supplement
C. Sodium choloride

6.What is the action /function of Ringer Lactate? A- Increase intracellular


fluids
B- Decrease intracellular fluids
Expend extracellular fluids ✅
Decreaseextracellularfluids

// 2996699650 ‫ ﻣﺣﻣد ﺳﺎﻣﻲ‬/‫أ‬

7.Patient during intraoperative period while administering anesethia. The patient complained
from reaction. The vital signs Bl. P 110/70 mmhg, HR 140 b/m, RR 24 b/m. Which of the
following drug should the nurse administer?
A. I. M epinephrine ✅ B. I. V atropine

8.What is the goal for giving Corticosteroids in Asthma?‫اذا ﺳﺋل ﻋن اﻟدواء ھذا ﻣن دون م ﯾﺣدد اﻻزﻣﮫ ﯾﻛون‬
‫ ﻟﻼﺟﺎﺑﮫ‬A
A. Bronchodilator
B. Decrease airway swelling ✅
C. Chest clear and Remove secretions
D. Vasodilation

9.Patient scheduled to perform venacava filter surgery. The doctor ordered to stop medication
before one day of operation for preoperative preparation. Which of the following drug should be
stopped?
Warfarin ✅
Sodium bicarb
Potassium

10.The Doctor prescribed 75 ml /hr of 0.9 NS. How much will infused through 12 hour?
A. 1500 B. 900✅ 75*12
11. C. 1000

11. The doctor ordered medication 5000 mg. The nurse deluted in 250 ml of D5N5 and modified
dose on infusion pump to 20mg/hr. How many ml should the nurse give per hour?
A. 5
B. 10 C. 15 D. 20✅

12.When administering medication to a four year-old child, which of the following actions should
a nurse take?
OA. Ask the child to take the medication
OB. Indicate to the child a firm expectation to take the medication
OC. Mix the medication with a large amount of the child's favourite food or drink ✅
OD. Inform the child that an injection will be necessary, if the child refuses to take oral
medication

13. A 55 year-old man takes cyclosporine mg by mouth twice per day. He had a heart transplant
two months ago. He complains of stomach cramping, diarrhea and muscle twitches. Which
electrolyte is most likely elevated?

// 2996699650 ‫ ﻣﺣﻣد ﺳﺎﻣﻲ‬/‫أ‬

a. Calcium
b. Sodium
c. Magnesium
d. Potassium

14.A patient was on a regular does of lithium carbonate. The nurse that he has hand tremor,
polyuria, diarrhea and vomiting. What immediate action should be taken by the nurse?
A. Diuretics
B. Withholding lithium
C. Calling the psychiatrist
D. Monitoring serum lithium level

15.A patient was on a course of lithium carbonate drug. During the


nurse found that he complained from nystagmus visual hallucination, and oliguria which of the
following drug related complications best ‫اﻟﺑول ﻗﻠﮫ‬
symptoms?
A. Overdose
B. Mild toxicity
C. Severe toxicity
D. Moderate toxicity

16.A 13-year-old patient is admitted for diarrhea and vomiting. He looks pale and lethargic. A
nurse is preparing to give IV hypotonic solution.

Blood pressure 110/70


Heart rate 76 /min
Respiratory 18/min
Temperature 39.1C

Which IV solution is most appropriate?


A. 0.9% saline
B. Lactated ringers
C. 10% dextrose in water
D. 0.45% sodium chloride

17. a nurse is caring for a patient who had Coronary Artery bypass Graft Surgery (CABG) four
hours ago. The nurse notices that the patient has increased confusion and is restless. The
patient reports nausea, weakness and paresthesia in the extremities (see lab results)

Test Result Normal Values


Sodium 145 134-146 mmol/L

// 2996699650 ‫ ﻣﺣﻣد ﺳﺎﻣﻲ‬/‫أ‬

Potassium 6.8 3.5-5.2 mmol/L


Calcium 2.50 2.15-2.62 mmol/L

Which of the following is the best medication? ‫اﻟﺑوﺗﺎﺳﯾوم ﻣرﺗﻔﻊ ﻋﻧده ﻋﺷﺎن ﻛذا م ﻧﻌطﯾﮫ ﺑوﺗﺎﺳﯾوم‬
A. Naloxone (Narcan)
B. Hydralazine (Apresoline)
C. Potassium chloride (KCI)
D. Sodium polystyrene sulfonate (Kayexalate)

18.Clomiphene citrate (Clomid) is prescribed for a 32-year-old infertility treatment the nurse
should understand that this medication is used for following actions?
A. Induce ovulation
B. Decrease prolactin level
C. Reduce endometriosis
D. Stimulate the release of Follicle-Stimulating Hormone

19.A nurse accidently dropped a medication ampoule, informed the charge nurse, and
completed an incident report form. The charge nurse arranges medication replacement.
Which of the following is the immediate nursing action required?
A. Revise protocol for medication related incidences
B. Allocate senior nurses to medication assignments
C. Provide missed medication dosage to patient first
D. Incident reporting must be given the priority

20. The nurse was giving drug to the patient it was tablet, the patient said that is not my
medication. Why my drug is changed in color? What should the nurse response??

A. Check the medication list

B. Check the doctor order ✅

C. Say this is your Medication

D. Say the doctor wrote new order new drug

21. Physician order three times the normal dose - when the nurse calls him, he insists on giving
the ordered medication - what does the nurse do?

A. Call pharmacy

B. Call another doctor.

C. Call nurse supervisor

Answer: C

// 2996699650 ‫ ﻣﺣﻣد ﺳﺎﻣﻲ‬/‫أ‬

22. Which medicine prescription the nurse should question?

A. Lasix 40mg oral BID


B. Codeine oral PRN‫ﻻﻧﮫ م ﻛﺗب اﻟدوز‬

23. Patient is receiving chemotherapy. When should the nurse give antiemetic to avoid side
effect?

A. Before session ✅
B. B. During session C. Half hour After session

24. What is drug that prevent recurrence of rheumatic fever?

‫ اﻋم ﻋﻼﻣﮫ ﻣن اﻋراﺿﮭﺎ اﻻﻟم اﻟﺟوﯾﻧت ﺑﯾن اھم ﺗﺣﻠﯾل ﻟﮭﺎ‬ASO

A-Penicillin ✅

B- Corticosteroids

C. Salicylates

Answer: A

25. Extrapyramidal adverse effects and symptoms are most often associated with which of the
following drug classes?‫ﻣﺟﻣوﻋﺔ ادوﯾﮫ ﻟﺳﺎﯾﻛو‬

A. Antidepressants

B. Antipsychotics

C. Antihypertensives

D. Antidysrhythmic

26-The nurse is caring for patient with asthma. What is the appropriate treatment for asthma?

A. Alpha antagonist, Corticosteroid

B.Beta antagonist, Corticosteroid

C. Alpha agonist, Corticosteroid

D-Beta agonist, Corticosteroid

‫أ‪ /‬ﻣﺣﻣد ﺳﺎﻣﻲ ‪// 2996699650‬‬


27.A 56-year-old man was admitted with complaint working for three days. The nurse is
preparing to administer infusion of saline 1000 ml over six hours. What is the hourly infusion
rate that the nurse needs to infusion therapy in mL/hour?1000\6

A.155

B.167

C.190

D.217

28. 500mg diluted in ml 250ml D5NS with infusion pump then she modified the dose to
20mg/hr.How many ml/hr. should the nurse give??20\500*250

A) 5

B) 10

C) 15

D) 20

29.A month baby how many ml orally he can receive?

A. 200

B. 400

C. 800

D.1200

30. The patient came to ER with second degree of burn on his dorsal left arm and dorsal of both
leg TBSA. The patient weight is 62 Kg. What is the total intravenous fluids the patient need!?

A. 5580 4.5+9+9*4*62=

B. 6580

C. 6680

D. 7680

// 2996699650 ‫ ﻣﺣﻣد ﺳﺎﻣﻲ‬/‫أ‬

31. Doctor order 1000 unit heparin in 5 ml.The available 5000 unit. How many ml should the
nurse give?1000\5000*5

A.1

B.2

C.3

32. Thenurse Assess Neonate after delivery using Apgar score. Spontaneous respiration(2)
Prompt respond(1) Limited cry (1) 98 pulse(1) Pinkish body color except hands(1) What is
the expected score ?

A— 8

B— 7

C— 6

D— 5

33 - A9 -year-old child is admitted to the Emergency Department injury. The child is oriented to
the place, person and time(4), opin eye (5) spontaneously, obeys commands(6). The nurse is
doing a ped Coma Scale (PGCS).Which of the following score the nurse should record?

A— 3 B— 8 C— 12 D— 15

34. The doctor order 0.2 g, the available 400 mg in quantity 10 ml, how many ml needed??

A-2.5ml

B-5ml

C- 2ml

D- 4ml

35. Physician order three times the normal dose - when the nurse calls him, he insists on giving
the ordered medication - what does the nurse do?

A. Call pharmacy

// 2996699650 ‫ ﻣﺣﻣد ﺳﺎﻣﻲ‬/‫أ‬

B. Call another doctor.

C. Call nurse supervisor

36. The nurse is caring woman that has cancer, and she is under Chemotherapy. She is
complaining anorexia and the patient has low weight. What should the nurse instruct
her???

A. Eat small meals every day.

B. Eat large meals every day.

C. Eat if you are hungry.

D. Eat your favorite food

37. What is the Contraindication of baby vaccine?

A. Antiemetic

B. Antibiotics

C. Steroid

38. The doctor order 600 ml of drug during 10 hours. How many minutes for 15 ml of the drug?

A. 15✅ 60*10=600 600\600*1

B. 20

C. 25

D. 30

39. Physician order three times the normal dose - when the nurse calls him, he insists on giving
the ordered medication - what does the nurse do?

A. Call pharmacy
B. B. Call another doctor.
C. C. Call nurse supervisor

// 2996699650 ‫ ﻣﺣﻣد ﺳﺎﻣﻲ‬/‫أ‬

40. Which medicine prescription the nurse should question?

A. Lasix 40mg oral BID


B. B. Codeine oral PRN

41.The Doctor prescribed 75 ml /hr of 0.9 NS. How much will infused through 12 hour?

A. 1500 B. 900
C. 1000

42. Girl come hospital with thalassemia . She is given blood transfusion every 3 weeks. She
came to hospital for blood transfusion and the doctor ordered lab investigation before
administeration, He found on results increase ferritin level.

What should the nurse give for her to prevent complication?


A. deferasirox
B. Iron supplement
C. Sodium choloride

43. Soldier was brought to triage area after being exposed to chemical weapons. Signs and
symptoms of nerve gas exposure were noticed. A nurse prepares for medical
management. Which medication should the nurse prepare for the patient?

A. Atropine
B. Adrenaline
C. Sodium nitrate
D. Sodiumthiosulphate

44. During a night shift a medical doctor complains of back pain and asks the t nurse to
give him morphine 5 mg IM. Which of the following actions indicates professionalism in
handing the ation by the nurse?
A. Call another doctor to manage
B. Refer him to Emergency Room
C. Administer morphine to doctor
D. Askhimtowriteaprescriptionfirst

45. Diabetic Patient with history of hypertension hyperthyroidism hyperlipidemia is


Going for a surgery. Which medication can be hold while the patient is NPO?
A. Lispro
B. Atorvastatin

// 2996699650 ‫ ﻣﺣﻣد ﺳﺎﻣﻲ‬/‫أ‬

C. Glucophage
D. Thyroxine

46.patient hospitalized then develop dysphagia and he has tablets, how to give him the
drug?
A. Dissolve it in water
B. Divide the pill in half and give him one half
C. Open the capsule and place it under his tongue
D. Looking for aliquid alternative to medication?

47.patient with liver disease and take lactulose enema. what type of enema? A-feeding
B-cleanes
C-medicated

48.Mastitis is an infection of the breast that occurs most often 2-4 after childbirth.
Which of the following is considered first line treatment of mastitis?
A. Drainage of breastabscess
B. Antibiotictherapy and cessation of breast feeding
C. Antibiotic therapy and continuation of breastfeeding e
D. Advise mother to stop breastfeeding until infection is clean.

49.The patient with gastroenteritis and he on I. V fluids N. S 500 ml starting at 13:00


pm. The doctor ordered the rate on infusion pump 125 ml/ hr. The nurse checked
the solution after 2 hours. She found empty bottle. The solution finished before timing.
What should the nurse do?
A. Check for Infusion pumps settings
B. Stop solution and start next at 17:00 pm
C. Administer new N. S now

50 .Which of the following MRSA medication?


A. Vancomycin
B. Gentamycin
C. Ampicillin
D. Streptomycin

51. What is the best time for corticosteroids drug intake ?


A. Before night sleep
B. Early morning 7am
C. After noon
D. 7pm

// 2996699650 ‫ ﻣﺣﻣد ﺳﺎﻣﻲ‬/‫أ‬

52. A nurse is preparing scheduled medications due at 6 pm. If a doctor orders


paracetamol tab 1g QID, and it was supplied from the pharmacy in 250 mg tablets.
Which of the following is the most appropriate nursing actions?
A- Ask the pharmacy to provide 1g tablets
B- Call the doctor to recheck the dosage
C- Give the patient four 250mg tablets
D- Hold the medication and document in nursing notes

53. A nurse is preparing to administer 25 mg iron dextran inject patient with iron
deficiency anemia .the nurse knows this d to subcutaneous tissue and wants to
administer the drug safely which of the best administration techniques ?
A.Z-track
B.deep im
C. use large gauge
D.insert needle at 45 angle

54- Quinine sulphate drug side effects:


A- Ringing in ears
B- Blindness
C- Hypotension
D- Insomnia

55. Child came to ER confirmed german measles. What is the best intervention for
child?
A. Antipyretics for low grade fever
B. Start I. V antibiotics C.Antihistamine for
itching

56.Which of the following MRSA medication?


A. Vancomycin ✅
B. Gentamycin
C. Ampicillin
D. Streptomycin

57.What is the best time for corticosteroids drug intake ?


A. Before night sleep
B. Early morning 7am ✅
C. After noon
D. 7pm

// 2996699650 ‫ ﻣﺣﻣد ﺳﺎﻣﻲ‬/‫أ‬

58-A nurse is preparing scheduled medications due at 6 pm. If a doctor orders


paracetamol tab 1g QID, and it was supplied from the pharmacy in 250 mg tablets.
Which of the following is the most appropriate nursing actions?
A- Ask the pharmacy to provide 1g tablets
B- Call the doctor to recheck the dosage
C- Givethepatientfour250mgtablets✅
D- Hold the medication and document in nursing notes

Maslow
56. Women come? to ER with her husband and the husband demonstrate she not
talk or voluntary eat since the son died
in accident, what the women need according to Maslow?
A. loving and belonging
B. psychological needs
C.spiritual support
D. family supports

57. According to Maslow hierarchy patient with schizophrenia has lack of motivation to
see herself. What are the needs for patient that lost?‫ﻓﻘدت اﻟﺛﻘﮫ ﺑﻧﻔﺳﮭﺎ‬
A. Physiological need
B. Self esteem
C. Love and belonging
D. Safet

// 2996699650 ‫ ﻣﺣﻣد ﺳﺎﻣﻲ‬/‫أ‬

Maternity

1. Post care recovery for mother of infection in vulva what is the important instructions for
discharge? ‫ﻋﻧدھﺎ ﻋدوى ف اﻟﻔل اش ﻧﻧﺻﺣﮭﺎ اذاطﻠﻌت‬
A. prevent recurrence of infection
B. maintain hygiene and prevent complications ✅

2 .Postpartum woman with normal vital signs and the fundus was boggy and soft. The nurse
performed massage for the fundus. The woman has been passed large amount of vaginal
blood. The fundus become firm. What is the next first action?‫اول ﺧطوه ﻋﻣﻠﻧﺎ ﻟو ﻣﺳﺎج اذا ظل ﻣﺛل م‬
‫ھوه ﻧﺳوي اﻟﺛﺎﻧﯾﮫ اﻓﺎﻟوﻛﯾت ﻟﻠﻣﺛﺎﻧﮫ ظل ﻣﺛل م ھو ﻧﻛﻠم اﻟدﻛﺗور‬
A. Notify the physician
B. Check vital signs
C. Remassage the fundus
D. evacoult bladder

3. her normal vaginal delivery. Her physical examination reveals a stable condition, breasts are
soft and her sanitary napkin has bright coloured rubra. Which of the following needs further
evaluation?‫وﻟدة وﻻده طﺑﯾﻌﯾﺔ ﻋﻣﻠﻧﺎ ﻓﺣص ﻟﺣﺎﻟﮫ ﻣﺳﻘره اﻟﺛدي طﻠﻊ ﺳوﻓت ودم اﻟﻧﻔﺎس ﻋﻧدھﺎ طﺑﯾﻌﻲ أي ﻣن اﻻﺟﺎﺑﮫ ﺧط ﯾﺗﺎج‬
‫اﻟﺑﯾﺷﻧت ﻟﺗﺛﻘﯾف‬
A. Amount and frequency of breast feeding
B. Hydration level and bleeding breast feeding
C. Activity, exercise and resting periods
D. Uterine size and position✅

4.Pregnant woman came to ER complain with sever pain in leg. When the nurse assessed her,
the nurse found that woman has varicose veins. What should the nurse do?‫ﻧﻧﺻﺣﮫ ﯾرﻓﻊ رﺟﻠﮫ ﻓوق‬
‫وﻛﻣﺎن ﯾﻠﺑس ﺷراب‬
A. Instruct her to Put pillow behind back while setting
B. Instruct her to put pillow under head and elevate the shoulder
C. Instruct her to wear elastic stocking with suitable size ✅
.

5.Postpartum Mother 2 days after delivery. Her baby Blood collection from heel is done to PKU.
She has appointment next day for circumcission. What is the most important attention for
circumcission?‫ ف أﻛﺗوﺑر‬19 ‫ﺳؤال‬
A. Avoid tighted daipper ✅ ‫ﻋﺷﺎن م ﯾﺻﯾر ﻟو ﻧزﯾف م ﺗﻛون ﺿﯾﻘﮫ‬
B. Notify first day of first voiding urination

6.Postpartum woman came to ER and complaining from fatigue . She told nurse that "I am so
exhausted, I am so worried, I am hoppeless and I can't do anything. The mother diagnosed with
postpartum blues ‫ م ﺗﮭﺗم ﺑطﻔﻠﮭﺎ‬. What is the next question The nurse should ask her ?‫اش ﻻزم اﺳﺄﻟﮭﺎ‬
A. Are you blame yourself you are not be coping to motherhood ? ✅ ‫ھل ﺗﺎﻗﻠﻣت ﻣﻊ اﻻﻣوﻣﮫ او ﻻ‬
B. Is there a family member you can talk with him?

// 2996699650 ‫ ﻣﺣﻣد ﺳﺎﻣﻲ‬/‫أ‬

C. Are you have any thoughts about harm yourself or harm your baby

7.Pregnant woman in active phase with cervical dilitation 3 cm. The midwife induced labor by
amniotomy. What is the next action? ‫ﺑﺗﺳﺎﻋدھﺎ وﺗﻔﺗﺢ ﻣوﯾﺔ اﻟراس‬
A. insertion catheter to empty bladder
B. Assess contraction for full minute
C. Assess FHR at least full minute ✅

8.The Vitro fertlization is related to (indications) ?‫ﺗﺧﺻﯾب اﻧﺑوﺑﻲ ﺗﻠﻘﯾﺢ ﺻﺎﻋﻲ ﻋﺷﺎن اﻟﻔﻠﯾﺑﯾون ﺗﯾوب ﻣﻘﻔﻠﮫ‬
A. Blockage of fallobian tube ✅
B. Donner of sperm
C. Absence of spearm
D. imnunolergichormon

9.Postpartum Client called to midwife after 10 days of discharge. She told her that lochia
become before 3 days brown. What is the most appropriate action?.What Inform her to return
hospital to reassessment ‫ﺑﻌد اﻟوﻻده ﺗﻐﯾر اﻟدم ﻧﻘول ﻟﮭﺎ طﺑﯾﻌﻲ‬
Tell her Thats normal and lochia changes in color from day to day ✅
Ask her about if she breastfeed her baby

10.What is the expected symptoms for uterine fibroid?


Irregular menstruation
Back pain
Pressure in
pelvic✅

11.A pregnant woman visits the Outpatient clinic complaining of excessive vaginal secretion.
Which of the following is the appropriate nursing assessment?
A. Fetal heart rate
B. Fundal height
C. Signs of infection of labor ✅
D. Fetalpresentationsandposition

12.A primigravida ‫ أول ﺣﻣل ﻟﮭﺎ‬mother is having her baby through normal vaginal delivery.
The baby is completely delivered but the mother is still experiencing the uterine
contractions.‫ﺑﻌد اﻟوﻻده ﻋﻧدھﺎ ﺗﻘﻠﺻﺎت وھﻲ ف اﻻﺳﺗﯾﺞ اﻟﺛﺎﻟث ﻻن ھﯾﺎ وﻟدة‬
A. Beginning of the third stage of labour ✅
B. Indication of increase in vaginal bleeding
C. Need for reducing the rate of intravenous oxytocin
D. Uterinecontractionwillgraduallyreducethensto

// 2996699650 ‫ ﻣﺣﻣد ﺳﺎﻣﻲ‬/‫أ‬

13. Why Postpartum hemorrhage cause infection?‫ﻟﯾش اﻟﻧزﯾف ﺑﻌد اﻻوﻻده ﯾﺳوي اﻧﻔﻛﺷن‬
A. Body not able to defense infection
B. Good media of bacteria ✅ ‫ﺑﯾﺋﮫ ﺟﯾده ﻟﻧو اﻟﻌدوى‬
C. Body loss blood component that help for prevent infection

14.A pregnant woman has previous preterm labor and she was asking the nurse if sexual
intercourse is safe during her pregnancy?‫وﻻده ﺳﺎﺑﻘﮫ ﻗﺑل وﻗﺗﮭﺎ ﺗﻘول ھل ﻓﯾﮫ اﺗﺻﺎل ﺟﻧﺳﻲ ﻧﻘول ﻋﺎدي ﺑس اﻟﺛﻠث اﻻﺧﯾر‬
‫ﻻ‬
A. Its safe
B. It‘s not safe
C. It‘s safe until 3rd trimaster ✅

15. What is Signs of Intrahepatic cholestasis of pregnancy?‫اﻟﺗﮭﺎب اﻟﻣراره اش اﻋراﺿﮭﺎ‬

A. Tachypnia
B. Intermittent tachycardia
C. Restlessness ✅ ‫)ﺣﻔظ ( ﻋدم اﻟراﺣﮫ‬

16.3 month Pregnant woman came with her husband to ER with fatigue and confusion. Her
husband inform the nurse that she doesn't have any disease . The doctor order maternal drug
screening test (drug abuse) Test for Her. What should the nurse do before the Test?‫ﺟﺎت ﺗﻘول اﻧﮭﺎ‬
‫ﺗﻌﺑﺎﻧﮫ وزوﺟﮭﺎ ﯾﻘول م ﻓﯾﮭﺎ ﺷﻲ واﻟدﻛﺗور ﯾطﻠب ﺗﺣﻠﯾل اﻻدﻣﺎن ع اﻟدوا ﻟﻣن ﻧﺟﻲ ﻧﺳوي ﻻزم ﻧﺎﺧذ ﻣواﻓﻘت اﻟزوج‬
A. Request for blood test
B. Get informed consent from husband ✅
C. Ask for medical history

17.A pregnant mother is in the active labour. Her cervix is fully effaced dilated with crowing of
the fetal head‫ اﻟﻔﺟﺎﯾﺎ ﻓل وراس اﻟﺟﻧﯾن طﺎﻟﻊ وﺗﻘول ﻣﻌد اﻗدر واﻟم ﺷدﯾد اش ﻧﻘول ﻟﮭﺎ‬. The mother is exhausted,
perspiring, and is having increasingly painful and severe uterine contractions. Which of the
following intervention is best ?desired by the assisting
?
A. exert pressure on abdomen to aid in child birth
B. increase the rate of intravenous medication
C. wipe face with wet towel and hold hands
D. instruct mother to push harder✅ ‫ﺷدي ع ﻧﻔﺳك ﻛﻣﺎن‬

18.A nurse in the labor room is performing a vaginal assessment on a pregnant client in labor.
The nurse notes the presence of the umbilical cord protruding from the vagina. Which of the
following would be the initial nursing action?‫ﻧﺳوي ﺗﻘﯾم ﻟﻠﻔﺟﺎﯾﺎﻧﺎ ﻧﻼﺣظﻧﺎ اﻧو اﻟﺣﺑل اﻟﺳوري ﻧﺎزل ﻣن اﻟﻔﺟﺎﯾﻧﺎ اش‬
‫ اﻟﺗدﺧل اﻻوﻟﻲ ﯾﻛون ﺑزوﺷن داﯾﻣﺎ‬initial
A. Place the client in Trendelen burg‘s position ✅
B. Call the delivery room to notify the staff that the client will be transported immediately
C. Gently push the cord into the vagina
D. Findtheclosesttelephoneandstatpagethephysician

19. When the bag of water ruptures, the nurse should watch for which of the following risks?‫اش‬

‫ﻧراﻗب اول ﻟﻣن ﻧﻔك اﻻﻣﻧﯾوﺗك ﻓﻠود‬


A. Fetal distress
B. Cord prolapse✅
C. Respiratory distress

// 2996699650 ‫ ﻣﺣﻣد ﺳﺎﻣﻲ‬/‫أ‬

D. Bleeding

20.30 years old women absent her menstruation for 5 months and her menstrual cycle come
every 28 days, she controlled her diet and do heavy exercise this women condition?‫اﻧﻘطﻌت‬
‫اﻟدوره ﯾﻌﻧﻲ اﻻﺟﺎﺑﮫ ﺳﻲ اذا م ﺟﺎﺗﮭﺎ ﻧﮭﺎﺋﯾﺎ ﻣن اﻟﺑﻠوغ ﯾﻌﻧﻲ ﺗﻛون ﺑﻲ‬
A. Pregnancy
B. Primary amenorrhea
C. Secondary amenorrhea ✅

21.A 7-year-old insulin dependent diabetic mother has delivered normally in 38 gestational
weeks. The nurse was assessing the insulin requirement this mother after delivery. What is the
insulin requirement for this patient?‫ﻛﺎن ﻣﻌﺎھﺎ ﺳﻛر اﻟﺣﻣل وﺗﺎﺧذ اﻧﺳﻠﯾن اش اﺳوي‬
A. Higher than before pregnancy
B. No changes in insulin requirement
C. Lower than when she was pregnant ✅ ‫اﻗﻠل ﺟرﻋﺔ اﻻﻧﺳﻠﯾن‬
D. Slightlyincreasedthanbeforedeliver

21. Which of the following should the midwife do for her episiotomy?‫ﺷق اﻟﻌﺟﺎن‬
‫اش اﻻﺟﺎﺑﺎت اﻟﻠﻲ ﺗﺳوﯾﮭﺎ‬
A. Mother her fetus with posterior position
B. Delayed Second stage of labor✅ ‫اذا ﻛﺎﻧت ف اﻻﺳﺗﯾﺞ اﻻﺛﺎﻧﻲ وﺗﻌﺳرت او دﺧل ﻣﺿﺎﻋﻔﺎت اﻧﺎ وش واﺳوي‬

11. Patient with yellowish sputum. What is expected lung sound?


A. Crackle
B. Wheezing
C. C. Bronchial

22. Multipara postpartum woman was experiencing hemorrhage. Which of the following
complication most common for her?
A. Infection✅
B. DVT

23. A 36-year-old son is the primary caregiver to his 76-year-old has many chronic diseases and
need full time assistance discussed with the community nurse the idea of referring his of the
elderly day care centers. The nurse explains the case to such services which of the following
elderly groups this patient belongs to this service?
A. With busy caregivers who need an assistance
B. Who are bored staying at home and need socialization
C. Who have been diagnosed with Alzheimer
D. Who want to engage in handcraft activities and art?

24. A 28-year-old pregnant woman at 9 weeks presents to the o with vaginal bleeding. During
assessment, the nurse found height is 12cm. Which of the following is the most likely
diagnosis? A-Placenta previa
B-Abruption placenta
C-Ectopic pregnancy

// 2996699650 ‫ ﻣﺣﻣد ﺳﺎﻣﻲ‬/‫أ‬


D-Hydatidiform mole

25. 20-year-old primigravida, who is pregnant at 40 weeks admitted to labor and delivery unit in
active labor. Vaginal revealed that, the fetal occiput is close to the maternal which of the
following would the nurse expect to be increase this fetal position?‫اي ﻣن اﻻﺟﺎﺑﺎت ﺗدل ع طﻠق طﺑﯾﻌﻲ‬
A. Leg cramps
B. Back discomfort
C. Vaginal bleeding
D. Nausea and vomiting

26.A 17-years-old mother presented to the primary health after delivery. She is suffering from
fatigue, anemia and fever vaginal discharge (see lab results)

Blood pressure 81/50 mmHg


Heart rate 98/min
Respiratory rate 26/ min
Temperature 39.6C

Test Results Normal values


RBC 4.6 4.7-6.1 × 1012/L
(Male)
L / 0104 × 4.2-2.4
(Female)
HB 88 130-170 g/L (Male)
Calcium 2.50 2.15-2.62 mmol/L g/L
061-041
(Female)

Which of the following is considered as the main maternal postpartum hemorrhage?


A. Death
B. Candidacies
C. Cervical cancer
D. Uterine prolapse

27.A nurse is performing an assessment of a women who is delivery is


which assessment finding would indicates a need to physician?

// 2996699650 ‫ ﻣﺣﻣد ﺳﺎﻣﻲ‬/‫أ‬

A. Hemoglobin of 11.0 g/dl


B. White blood cell count of 12,000
C. Fetal heart rate of 180 beats per minute
D. Maternal pulse rate of 85 beats per minute

28. A mother in the postnatal Ward is being provided with the her family and she has been
instructed to be on the bed members believe that the diet with the specific home increase the
milk production and will promote her healing Which of the following intervention is the most
desired?
A. Respect the cultural believes and encourage activities
B. Teach and monitor for correct breast feeding practices
C. Discuss dietary plan and monitor weight gain
D. Explain post-partum care and follow ups

29) A women was discharged from gynecological ward after gestational trophoblastic disease
(molar pregnancy) which of the following is the best advice to give her?
A. Never to fall pregnant again
B. To request the doctor to sterilize her
C. To consider having her uterus removed
D. To avoid falling pregnant for at least one year‫ ﻋﻣﻠﯾﺔ ﺗﻧظﯾف‬D and c

30. Which of the following statement described the latent phase?


A. First 3 cm of cervical dilatation
B. From onset of labor to full dilatation
C. When the cervix is 100% effaced
D. Time of progress from 4 cm to 7 cm

31. the nurse was educating a postpartum woman during discharge about importance of breast
feeding which of the following if said by the women, indicates the need for further education?
A. Breast milk is nutritionally balanced
B. Breast milk reduces the risk of infection
C. Breast feeding promotes mother-child bonding
D. Breast feeding prevents pregnancy

23-When the nurse assessed the fundus of a multiparous mother who delivered 2 hours ago,
she found the following. Level: 2 cm above the umbilicus Position: deviated to the right.
Consistency: Not well contracted What is the next nursing action after massaging the fundus
until it Becomes firm?
A. Assess vital signs
B. Increase V fluids
C. Evacuate the bladder
D. Ask the mother to rest

// 2996699650 ‫ ﻣﺣﻣد ﺳﺎﻣﻲ‬/‫أ‬

33. A 33-year-old patient in her 13th week of pregnancy has come for her routine gynecological
Appointment. According to the ultrasound, her fetus is a female. She already has three
daughters but her family wants the male child, so she asked the nurse guidance to terminate
her pregnancy. The Nurse explained that baby's gender is determined by the father. She later
involved her husband in the Discharge planning and finally helped the father agree to
continue the mother's pregnancy.
What professional obligation is fulfilled through the nursing implementation
A. Influenced decision making
B. Incident prevention beforehand
C. Empathetic leadership approach
D. Sympathetic knowledge sharing

34. A laboring woman desires to participate in her cesarean del pain control. Which of the
following methods would satisfy the woman's?
A-Epidural block
B-Pudendal block
C-Meperidine injection
D-General anesthesia

35. A 23-year-old vaginal delivery primigravida mother was discomfort due to breast
engorgement on the second post The mother complained of pain on the breast site and the
able to suck the milk. Which of the following will relief the mother's discomfort?‫وﺣده ﻋﻧدھﺎ ﺗﺻﻠب‬
‫ف اﻟﺛدي‬
A-Breast binder
B-Well-fitting brassiere
C-Encourage breast feeding
D-Lactation suppressing medication

36. After 3 days of breast feeding a post partial patient reports nipple soreness. To relieve her
discomfort the nurse should suggest that she:
A-lubricate her nipples with expressed milk before feeding.
B-dry her nipples with soft towel after feeding.
C-Apply warm compresses to her nipples just before feeding.
D-Apply soap directly to her nipples, and then rinse.

37.which of the following condition is a Contraindication for a woman oral


Contraceptives?
A. Dysmenorrhea
B. Menorrhagia
C. Thrombophlebitis
D. Toxic shock syndrome

38.A group of mothers, in their third trimesters, are attending the education session regarding
the alert signs for which they are immediately contact the hospital. The midwife is to assess the

// 2996699650 ‫ ﻣﺣﻣد ﺳﺎﻣﻲ‬/‫أ‬

understanding of the session by asking question. What response from the mothers helps assure
the midwife about right understanding?
A. Gush of urinary outflow without any urge
B. Low back pain at night and difficulty in sleep
C. Activity intolerance and breathlessness on exertion
D. Absence of contraction after 42 weeks of pregnancy

39.A 25-year-old mother gravid 2 para q came for a routine check Antenatal Clinic. The nurse
assessed fetal heart rate for the 38 pregnant mother. What is the expected normal fetal heart
rate per minute?
A. 90
B. 100
C. 140
D.170

40. a primigravid women attends at the outpatient clinic for up. History taking revealed that she
is on medical hypothyroidism. The nurse has classified this pregnancy one. What is the most
possible complication lead to this class if?‫ ﻧﺳوي ﺗﺣﻠﯾل‬tch ‫اذا ﻛﺎن اﻟﻧﺷﺎط ﻗﻠﯾل ﯾﻛون ﻋﺎﻟﻲ واﻟﻌﻛس‬
A. Preeclampsia
B. Preterm labor
C. Congenital anomalies
D. Postpartum hemorrhage

41. A primigravida mother is assisted out of the bed a few normal vaginal delivery. She is taken
to the bathroom cleaning herself and to pass urine. She has difficulty in commode seat and
is having no urge to urinate. Which of the following intervention is the most desired?
A. Teach kegel's exercises
B. Give warm sits bath first
C. Pour warm water over vulva
D. Identify possible perineal tears

42) A 65-year-old women visited the gynecological outpatient history reveals that she had 3
pregnancies, one abortion gestational age, had 2 normal deliveries. She smokes 20 Her
complaint is that she wets herself when she cough embarrassing for her? Which of the following
can be considered as risk factors pelvic floor muscles?
A. Chronic coughing
B. Diabetes mellitus
C. Excessive spot
D. Sedentary life style

43. A 22-year-old gravida 2 para 1 with gestational age 38 week admitted to the hospital. The
chief complaint is decreased the fetal non-stress test revealed decreased variability and fetal
movement. The next morning as part of the antenatal the nurse checks the fetal heart rate by
Doppler Sonicaid decreased the fetal heart rate to less than 100 /min. which of the following
action the nurse should do first?

// 2996699650 ‫ ﻣﺣﻣد ﺳﺎﻣﻲ‬/‫أ‬

A. Reassure the mother that the FHR is Ok


B. Notify immediately the physician or midwife
C. Reposition the patient to left lateral position
D. Ask the mother about the pattern of fetal movement

44.A 23-year-old vaginal delivery primigravida mother was discomfort due to breast
engorgement on the second post The mother complained of pain on the breast site and the able
to suck the milk. Which of the following will relief the mother's discomfort?
A. Breast binder
B. Well-fitting brassiere
C. Encourage breast feeding
D. Lactation suppressing medication

45. Which of the following is the appropriate nursing advice to mother complaining of epigastric
burning sensation and do use of drugs?‫ﻋﻧده زﯾﺎده ف ﺣﻣض اﻟﻣﻌده ﻧﻘوﻟﮫ ﯾﺑﺗﻌد ﻋن اﻟﻔﻼﻓل وﯾﺎﻛل وﺟﺑﺎت ﺻﻐﯾره‬
A. Chewing gums
B. Increase fluids at bedtime
C. Drink orange juice on getting up
D. Eat small meals every 2 to 3 hours

46. When performing a postpartum assessment on a women the


presence of clots in the lochia‫دم اﻟﻧﻔﺎس ﻣﻣﻛن ﯾطﻠﻊ ﻣﻌﺎه ﺗﺟﻠطﺎت اذا ﻛﺎﻧت اﻛﺛر‬
‫ ﻣن واد ﻧﺑﻠﻎ اﻟطﺑﯾب‬. The nurse examines the that they are larger than 1
cm, Which of the following nursing actions is most appropriate?
A. Document the findings
B. Notify the physician
C. Reassess the client in 2 hours
D. Encourage increased intake of fluids

47. A 33-year-old women has come to the outpatient clinic for treatment of a vaginal infection.
Physical assessment reveals yellowish excessive, thin offensive and frothy discharge.
Which of the following is the most likely diagnosis?
A. Candidiasis
B. Trichomoniasis
C. Bacterial vaginosis
D. Chlamydia

48. A 17-years-old mother presented to the primary health after delivery She is suffering from
fatigue, anemia, fever vaginal discharge (see lab results)
Blood pressure Heart rate Respiratory rate Temperature
Test Result. RBC 4.6
Hb 88
81/50 mmHg 98 /min
26 /min
39.6 C
Normal Values

// 2996699650 ‫ ﻣﺣﻣد ﺳﺎﻣﻲ‬/‫أ‬

(Male) (Female)
(Male) (Female)
4.7-6.1 × 1012/L 4.2 - 5.4×1012/L
130-170 g/L 120-160 g/L
Which of the following is considered as the main maternal Postpartum hemorrhage?
A. Death
B. Candidacies
C. Cervical cancer
D. Uterine prolapse

49. A mother, who is planned for the labor induction, is started on intravenous medication. She
is in the first stage of her labor and is having regular and increasingly stronger uterine
contractions her cervix is1cm dilated for the past few hours; both the mother and the
baby are being monitored.
Which of the following signs should alert the midwife?‫أي ﻣن اﻻﺟﺎﺑﺎت ﺗﻛون اﻟﻘﺎﺑﻠﮫ ﯾﻘﺿﮫ‬
A. Baby's head not engaged
B. Decreasing heart rate of the baby
C. Mother's blood pressure 110/60 mmHg
D. Mother's perspiration and increased thirst

50) A postpartum mother is to be discharged on the second day of her forceps delivery. She
had sutures on her vaginal‫ ﻋﻧده ﺧﯾﺎطﮫ ف اﻟﻔﺟﺎﯾﻧﺎ‬and perineum tears. She is breastfeeding her
baby and eating the special food provided by her family. What discharge teaching needs
more emphasis?
A. Diet management and exercise plan
B. Newborn care and vaccination records
C. Hygiene practices and alert signs to report
E.Family planning and child growth
monitoring

51. The Postpartum mother was asking the nurse about timing for restating sexual

intercourse activity. What should the nurse response?‫ام ﺑﻌد اﻟوﻻده ﺗﺳﺎل ﻋن وﻗت اﻟﺟﻧس‬

‫ﺑﻌد ﺗوﻗف اﻟﺣﯾض‬

A. 3 weeks after delivery

B. As long as taking contraceptives

C. After stop of lochia discharges✅

D. Any time she wants

‫أ‪ /‬ﻣﺣﻣد ﺳﺎﻣﻲ ‪// 2996699650‬‬


52. A13-weeks-pregnant, multigravida women is anxious, and apprehensive she

has five children and is not.willing to continue with this pregnancy she is

requesting the midwife to about the fetus, she is underweight, malnutrition and is

over worked, BMI 17kg/m2, what intervention is desired immediately?

A. Admission and intravenous line management for induction.

B. Family planning and birth control measures

C. Dietary management and supplements.

D. Support, reassurance and counselling. ✅

53. A nurse is assessing the uterus of a G5P4 patient immediately after delivery.

The Nurse notes the fundus is not contracted. Which of the following is the most

appropriate immediate action should be taken? A. Massage the fundus B. Assess

the bladder C. Elevated the mother's legs D. Encourage the mother to void

54. pregnancy patches contradiction?

‫ﯾﺣطول ﻟزﻗﮫ ﻣﺎﻧﻌﮫ ﻟﻠﺣﻣل‬

A-Over wright 80 ✅

B-Have diarrhea.

C- With antibiotic

55. Pregnant woman came to ER with rupture of membrane. Why should the
nurse Limit vaginal examination for her?

A. Prevent risk of infection✅

B. Avoid bleeding C. Prevent further loss of membrane fluid D. Prevent fetal


hypoxia

‫أ‪ /‬ﻣﺣﻣد ﺳﺎﻣﻲ ‪// 2996699650‬‬


56. The pregnant woman is asking the nurse about the function of placenta. The

nurse explains to her that placenta provide your baby with food and Oxygen.

Which another:-

A. Cushion and protect baby. B. Produce HCG C. Produce contraction

57. A 20 weeks pregnant, primary gravid woman visits the antenatal has sickle cell

anemia trait and worried this disease transmitted to her baby which of the

following should be initial intervention?‫اش اﻟﺗدﺧل اﻻوﻟﻲ‬

A. Plan for the fetal genetic screening

B. Educate mother that her disease is inactive.

C. Discuss the chances of genetic disease in the fetus.

D. Gather data about the other family members having the disease.

58. he Oral contraceptives increase risk for any cancer for long-term using??

A. Cervical cancer

B. Ovarian cancer

C. Skin cancer

D. Breast cancer

59. pregnant women G1 P0 vaginal delivery observed in the second postpartum

day that the perineal pad. saturated with bright red lochia rubra what is the

priority nursing intervention?

A. Massage fundus

B. Obtain vital signs.

C. Inform physician

D. Inquire about time of pervious saturated perineal pads.

// 2996699650 ‫ ﻣﺣﻣد ﺳﺎﻣﻲ‬/‫أ‬

60. Full term Newborn was delivered 2 hour ago. The nurse checks him observed.

mild peripheral cyanosis. What is the first action?

A. Call for help

B. Notify the doctor

C. Check temperature

61. Pregnant mothers in 9 weeks she came for ER. with vaginal bleeding and

lower abdominal cramp ...on U/S ...the concept products in lower uterus‫اﻧواع‬

‫ اﻻﺟﺎض‬A-B-C-D

A. Missed abortion ‫ﻓﺎﺋت‬

B. Threaten abortion‫ﻣﮭدد‬

C. Inevitable abortion‫ﻻﻣﻔر ﻣن‬

D. Complete abortion‫ﻣﻛﺗﻣل‬

62. Pregnant mothers in 9 weeks she came for ER. with vaginal bleeding and

lower abdominal

cramp ...on U/S ...the concept products in lower uterus

A. Missed abortion

B. Threaten abortion

C. Inevitable abortion

D. Complete abortion

63.The nurse examined Pregnant woman for the five factors affecting labor process. The five Ps
(passenger, passage, powers, placenta, and .....? Which of the following additional factor?
A. Pathway
B. Psychology ‫✅ﺣـﻔظ‬
C. Process

// 2996699650 ‫ ﻣﺣﻣد ﺳﺎﻣﻲ‬/‫أ‬

64. 45years old Woman with fibroid‫ورم ﻟﯾﻔﻲ‬. The doctor prescribed for her Gonadotropin
releasing hormone‫( اﻓراز ھرون اﻟﻐدد اﻟﺗﻧﺎﺳﻠﯾﮫ‬GnRH) agonists. What is the side effect for hcg
hormone?

A. Depression
B. Anorexia
C. Osteoarthritis
D. Menopause

65. The nurse is instructing a female client how to do breast self-exam. Which of

the following is the best time to perform this exam?

A. After ovulation
B. After period
C. Two weeks after period
D. Three days before period

66. A 17-year-old mother presented to the primary health center ten after delivery.

She is suffering from fatigue,anemia, fever and vaginal discharge (see lab results) Blood

pressure 80/50 mmHg Heart rate 112 /min Respiratory rate 35 /min Temperature

39.6 C Test Result Normal Values RBC 4 4.7-6.1 × 1012 /L (male) 4.2-5.4 × 1012

/L (female) Hb 90 130-170 g/L 120-160 g/L (female) HCT 0.29 0.42-0.52 (male)

0.37-0.48 (female) WBC 12.8 4.5-10.5 × 109/L Which of the following is the best

diagnosis of health problem in this case?‫ﯾﻛون ﻋﻧدھﺎ ﺣﻣﻰ اﻟﻧﻔﺎس‬

A. Severe urinary track infection


B. Vesical-vaginal fistula
C. Puerperal sepsis
D. Post-partum hemorrhage

67. What is hormones for biological difference between male and female?

A. Estrogen and HCG


B. Testosterone and progesterone

// 2996699650 ‫ ﻣﺣﻣد ﺳﺎﻣﻲ‬/‫أ‬

68.The Vitro fertilization is related to (indications) ?

A. Blockage of fallopian tube


B. Donner of sperm
C. Absence of sperm
D. immune allergic hormone

69.What is the characteristics of ectopic pregnancy pain

A- Abdominal cramp

B- Throbbing pain in lower part


‫أﻟم ﻣﺛل اﻟﻧﺑض‬
C- Sudden pain in upper part

70. A mother is in the midwife's clinic with the complaint of, redness and swelling in

her right breast. She is breast feeding her nine months-old baby. What should be
immediate intervention?
A. Start on antibiotics
B. Detailed investigations
C. Stop the breast feeding.
D. Wound cleaning and dressing

71 _ Which hormone is release from placenta?

A. HCG
B. Progesterone
C. Estrogen

.72. What is the benefit of Kegel exercise?

A- Increase strength of pelvic muscle


B- Induce cervix contraction

73. Which of the following statement indicate nursing action during the first hour

after delivery of the placenta?

A. Monitor of mother‘s hemoglobin


B. Assess maternal vital signs every 15 minutes
C. Ensure that the mother mobilize and empty her bladder
D. Administer 10 units ofoxytocin via IV linetoensure uterus is wellcontracted.

// 2996699650 ‫ ﻣﺣﻣد ﺳﺎﻣﻲ‬/‫أ‬

74.A nurse is taking care of a woman who is in active labor. She is multipara at 38
weeks gestation. Her cervix is 5 cm dilated at -2 station with ruptured membranes.
On the fetal heart rate tracing, the nurse observes a fetal heart rate baseline of 150
beats per minute with Deceleration in fetal heart rate to as low as 100 beats per
minute. Deceleration in heart rate occurs both during and between contractions. With
each deceleration heart rate quickly returns to baseline after a few seconds. The nurse
has anticipated which of the following.?
A. Client will probably deliver quickly.
B. Client may need rapid intervention for a prolapsed cord.
C.Client has a normal fetal heart rate tracing for a multipara in active labor.
D. Client has a fetal heart rate tracing consistent with poor uteroplacental blood

75. A group of mothers, in their third trimesters, are attending the education session
regarding the alert signs for which they are immediately contact the hospital. The
midwife is to assess the understanding of the session by asking question.>>> What
response for the mothers helps assure the midwife about right understanding?
A— Gush of urinary outflow without any urge
B — Low back pain at night and difficulty in sleep
C— Activity into leranceand breath lessnesson exertion
D — Absence of contraction after 42weeks of pregnancy

76. Which of the following statement indicate nursing action during the first hour
after delivery of the placenta?
A. Monitor of mother‘s hemoglobin
B. Assess maternal vital signs every 15 minutes
C. Ensure that the mother mobilize and empty her bladder
D. Administer 10 units ofoxytocin via IV linetoensure uterus is well contracted.

77. what is the antenatal assessment schedule for woman the period of
conception and 28 weeks of gestation?
A. once a week
B.Every 2 week
C. Every 3 week
D.Every 4 week

78. A woman is at 30 weeks gestational age admitted to antenatal with premature


rupture of membrane . the nurse administered Dexamethasone to her according doctor
prescription. She asks what is the Drug for. Which of the following the best answer?
A. To promote fetal lung maturation
B. Prevention of chorioamnionitis
C. To increase uteroplacental exchange
D. Treatment of fetal respiratory distress

// 2996699650 ‫ ﻣﺣﻣد ﺳﺎﻣﻲ‬/‫أ‬

79. 25-year-old woman has a family history of breast cancer. The nurse reviews the
procedure for breast self- examination (BSE) and tells her that the best time for a woman
to perform a breast self-examination is:
A. a few days before her period.
B. during her menstrual period.
C. on the last day of menstrual flow.
D. 3- 7days after the beginning of her period

80. A 29-year-old woman had been diagnosed with a 3 cm ovarian cyst.Which of


the following is the appropriate step in management?
A. Cyst aspiration
B. Hormonal therapy
C. Cyst removal by laparoscopy
D. Examination after next menstruation

81-Why does hypertonic uterine contractions cause fetal hypoxia ‫اﻟﻧﻘﺑﺎﺿﺎت ﺗﺳﺑب ھﯾﺑواﻛﺳﯾﺎ ﻟطﻔل‬
‫ﻋﺷﺎن ﺑﯾﺿﻐط ع اﻟﺣﺑل اﻟﺳري‬
A. It compress the cord and prevent exchange of oxygen wastes ✅
B. It interrupts placental blood flowǁ

82-.Postpartum mother after 4 days of Cesarean Section. She has BMI 29. The nurse
wants to give health instructions for her. Which of the following teachings should the
nurse provide about ?
A. Infection
B. Puerperal sepsis
C. Diet and exercises ✅
D. Bleeding

83-After delivery estrogen and progesterone drop down there is increase in


which hormone ?‫ھذي اﻟﮭرﻣوﻧﯾن ﺗﻧﺧﻔض ﺑﻌد اﻟوﻻد وﯾﺗرﻓﻊ اﻟﺑروﻻﻛﺗﯾن‬
1. FSH
2. LH
3. prolactin✅

84-Pregnant women dignoised with herpis wich is the best mothed for delivery ?
1-cesarean section✅ ‫ھﯾرﺑس‬
2- normal delivery
4. forceps

85. Postpartum Mother delivered since 5 hours. She complained of pain from breast
feeding. She never experienced pain before. What should the nurse instruct her?
A.Stop breast feeding
B.It is Emergency

‫أ‪ /‬ﻣﺣﻣد ﺳﺎﻣﻲ ‪2996699650 //‬‬


C.Normal and take pain medication ✅

86. Postpartum Mother was complaining from hemorrhage. The bleeding controlled and
stop 4 hours ago. The mother is asking now the nurse to go bathroom for urination.
What should the nurse response?
‫ﺑﻌد اﻟوﻻده ﺗﻌﻧﻲ ﻣن ﻧزﯾف ﻣﺗوﻗف ﻣن ارﺑﻊ‬
‫ﺳﺎﻋﺎت وھﻲ ﺗﺑﻐﻰ ﺗروح اﻟﺣﻣﺎم وش ﺗﺳوي‬
‫ﺗﻘوم ﺑﺷوﯾش و ﺗﻧزل رﺟﻠﮭﺎ ﺷوي ﺷوي‬
A.Don't let her go to bathroom
B. Let her to go bathroom
C. Instruct mother to set on the bed then put her leg dow
gradually
D. Keep her in bed and Provide bed pan

87. Mother want to use hormonal patch as contraceptive device. She is obese. What will
reduce the effect of this device?
A.Weight over 80 ✅
B.Using Antibiotic C.Age less than 35

88. Women with bipolar taking carbamazepine. Which of the following side effects?
A-False positive pregnancy test
B- Sexual dysfunction

‫أ‪ /‬ﻣﺣﻣد ﺳﺎﻣﻲ ‪// 2996699650‬‬


Infection control
1. What is the transmission method of HAb virus?
A. Food ✅
B. Blood
C. Sexual transmission

2. The nurse is assessing a 65-year-old patient, who reports the fatigue, weight loss, night
sweats, and a productive cough with thick sputum The nurse should immediately initiate
isolation precautions for which of the following?
A. Influenza
B. Pertussis
C. Bacterial pneumonia
D. Pulmonary tuberculosis

3. The nurse is assessing a 65-year-old patient, who reports the fatigue, weight loss, night
sweats, and a productive cough with thick sputum The nurse should immediately initiate
isolation precautions for which of the following?
A. Influenza
B. Pertussis
C. Bacterial pneumonia
D. Pulmonarytuberculosis✅

4.A 33-year-old man with meningitis present Department. Healthcare providers plan to
precautions what is the best nursing practice for this case?
A. Use N95 mask all the time
B. Monitor the negative air pressure
C. Use respiratory protective equipment all the
D. Keep the patients with the same diagnosis

5.a nurse receives a telephone call from the admission office of the hospital and is told that a
patient with streptococcal meningitis will be admitted to the Medical Unit. The nurse is planning
to apply infection control measures For the patient which type of isolation precaution the nurse
must observe?
A. Droplet precautions
B. Contact precautions
C. Airborne precautions
D. Standard precautions

// 2996699650 ‫ ﻣﺣﻣد ﺳﺎﻣﻲ‬/‫أ‬

6-To reduce the risk of transmitting methicillin-resistant Staphylococcus Aurous (MRSA) from an
infectious wound, which of the following Precautions should be implemented?
A. Airborne
B. Contact
C. Droplet
D. Reverse isolation

7.A 45-year-old patient admitted with pulmonary tuberculosis nurse placed the patient in an
isolation room with negative and prepared all the personal protective equipment at the ent room.
What type of precaution measure has the nurse activated?
A. Contact
B. Droplet
C. Airborne
D. Standard

8.A nurse is planning to discharge a known HIV, Which of the following nursing problem
requires?
A. Risk of infection due to altered immune
B. Fluid volume deficit due to frequent diarrhea
C. Anxiety due to disease, fear and social
D. Weight loss due to higher metabolism rate

9.A nurse is preparing a sterile field prior, which of the following procedures illustrates the nurse
to the principles of aseptic technique?
A. Touch the outer 2-inch border of the sterile gloves
B. Place the sterile linen-wrapped package on waist level
C. Hold the bottle
D. Open sterile item while holding outside hand then throw object onto sterile field

10 . A 47-year-old man is diagnosed with tuberculosis (TB) and negative pressure room which
of the following should wear a face mask? ‫اﻟﻧﺎس ﺗﺑﻐﻰ ﺗزوره اﻟﻛل ﯾﻠﺑس ﻣﺎﺳك‬
A. Patient's health care providers
B. All people who enter the patient's room
C. Person has close contact with the patient
D. Family members who are at risk for infection

11. A patient returns to a clinic for a follow-up visit and is diagnosed as positive for Human
Immunodeficiency Virus (HIV). The patient expresses fear related to lack of finances, fear of
social avoidance, and hopelessness. Which of the following nursing action should provide the
most patient support?
A. Referral to a physician infectious disease specialist
B. Referral to a community-based HIV clinic
C. Referral to the local public health department
D. Recommendation to disclose the diagnosis to family

// 2996699650 ‫ ﻣﺣﻣد ﺳﺎﻣﻲ‬/‫أ‬

12.A nurse working in medical unit is preparing to with droplet precaution measures in place.
The the following personal protective ‫ﻋزل‬
equipment Eyewear, what is the correct sequence foe putting the
equipment on?
A. Face Mask, Gown, Eyewear, and Gloves
B. Gown, Face Mask, Eyewear, and Gloves
C. Eyewear, Cloves, Face Mask, and Gown
D. Gloves, Gown, Face Mask, and Eyewear

13. A patient, who had abdominal surgery six days ago, has been ambulating in The halls
without much difficulty. however, on seventh postoperative day, the patient complains of
increased pain at incisional site and is walking bent over. What is the most likely
cause?

A. Intestinal inflammation
B. Pulmonary edema
C. Wound infection
D. Deep vein thrombosis

14. Patient with smallpox. He became fine and no symptoms but still isolated for
12-14 days. What is the type of the period?‫م ﻋﻧده أي ﻋﻼﻣﮭﻣﻌﻧﺎﺗﮫ ﻓﺗرة ﺣﺿﺎﻧﮫ‬
A. Incubation period
B. Infective period
C. Latency period
D. Communicable period

15. When the nurse starting change dressing procedure. What is the first step for
her?
A. Wear mask
B. B. Wear gown
C. C. Sterile gloves✅

16. Woman has experienced recurrent vaginal infection and she is asking the
nurse how to prevent the infection. What is the response if she says the nurse expect
she need further instructions and consider alter preventive measures that kill good
bacteria?
A. Wear cotton untied underwear
B. Wipes front to back each toilet.
C. Wear condom to prevent infection.
D. Avoid douche and perfume spray

17. What precautions are necessary when caring for a patient with Hepatitis A to
protect family from infected?
A. wash dish and do not share.

// 2996699650 ‫ ﻣﺣﻣد ﺳﺎﻣﻲ‬/‫أ‬

B. not sharing the toilet.


D. Placing the patient in a private room.

18. Recurrent urinary tract infection what advice to give?


A. Drink2-3litersofwater
B. Wipe from back to front
C. Urinate only when you have the urge.

19. Measurements rates of epidemiology. rate of birth in the year of 2016 is 19.5
per 1000. What is it considered?‫ﺗﻌرﯾف ﻻﯾش‬
A-Crude rate
B- Observation rate
C- Specific rate
D- Standardizedrate

20. HAb1transmission method?


A. Blood
B. Sexual
C. transmission
D. food

21. Woman has experienced recurrent vaginal infection and she is asking the nurse
how to prevent the infection. What is the response if she say the nurse expect she need
further instructions and consider alter preventive measures that kill good bacteria?
A. Wear cotton untied underwear
B. Wipes front to back each toilet.
C. Wear condom to prevent infection.
D. Avoid douche and perfume spray

22.most common bacteria UTI?


A. E. coli
B. Streptococcus asb

23. Patient with fever, cough with blood. The nurse call to infection center to provide
precaution to the patient. Which precaution should the nurse Apply?
A—Airborne
B— Droplet
C—Contact
D— Precaution until the confirm of diagnosis

34. HIV patient will be risk for which life threating disease?
A. Pneumonia
B. Tuberculosis

// 2996699650 ‫ ﻣﺣﻣد ﺳﺎﻣﻲ‬/‫أ‬

35.The nurse gave a prescribed intramuscular medication to the patient during a home health
visit. How should a nurse dispose of a used needle and syringe?

A) Recap the needle, then place the needle and syringe into a waterproof.
container until safe disposal can be made .
B) Bend the needle back towards the barrel of the syringe before putting . the
needle and syringe in a metal trash container
C) Wrap the needle and syringe in disposable paper before putting the needle
and syringe into the dirty section of the nurse's equipment bag
D) Put the needle and syringe directly into a puncture - resistant Plastic
container that has a lid

36-A male patient has an HIV and has a wife and son , his wife is depressed due to his
diagnosis What's the immediate goal ?
A.Risk for infection related to low immunity
✅ B.Risk for infective psychosocial support

37-The hospital observed MERS-COV ‫ ﻛوروﻧﺎ‬infection spreed to many persons. The


hospital not identified the affected from non-affected person‫م ﻋﻧدھم ﻋﻠم ﻣن اﻟﻣﺻﺎب ﻣن ﻏﯾر‬
‫ اﻟﻣﺻﺎب‬. What should do to reduce the risk?
A. Isolate all patients
B. Prevent visiting
C. Apply standard precautions to all patients ✅ ‫ﺗﺗﻌﺎﻣل ﻣﻌﺎھم ﻛﻠﮭم ع اﻧﮭم ﻣﺻﺎﺑﯾن ﻻﻧﮭم ﻣﺟﮭوﻟﯾن‬
D. Wear protective equipments

// 2996699650 ‫ ﻣﺣﻣد ﺳﺎﻣﻲ‬/‫أ‬

Fundamental and adult


1 . The patient came to ER with hip fracture. After surgery she is high risk for DVT. To prevent
dvt she wear anti embolism stockings .What‘s the additional instruction nurse needed to focus
on ?‫ﻣرﯾض ﻧﺧﺎف ﻋﻠﯾﮫ ﻣن ﺟﻠطت اﻟﻘدم ﻧﻐﯾر اﻟﺑزوﺷن‬
A. reposition pt frequently ✅
B. assess calves for warmth and tenderness

2. Patient with pacemaker. What is the discharge instructions should the nurse give?‫ﺣطﯾﻧﺎ ﻟو ﻣﻧظم‬
‫ﻟﻧﺑﺿﺎت ﻟﻠﻘﻠب وش ﻧوﺟﮭﮫ‬
A. Report doctor if doing heavy activity or carry more than 20 kg
B. Inform doctor if the ring out reading of normal rate ✅
C. Do light exercise

3 Patient came to ER with cold clammy skin and loss of hair in leg. He has pressure ulcer in
both leg. What is the most appropriate nursing diagnosis?
A. Risk for immobility
B. Risk for injury ✅
C. Risk for fall

4.What is the point of maximum pulse monitoring while auscultated apical pulse?
A. 5th intercostal space left midclavicular ✅
B. 5th right
C. 4th left
D. 4th right

5 Patient with cardiac disease. What is the expected symptoms?


A. Tachypnea, bradycardia and fever✅
B. Muffled heart sound
C. Cyanosis in extremities

.6 Q. Which of the following measures can reduce or prevent the incidence of atelectasis in a
post-operative client?‫اﻟرﺋﮫ ﻣﻧﻛﻣﺷﮫ ﻋﺷﺎن ﻧﺧﻔﺿﮭﺎ ﺑﻌد اﻟﻌﻣﻠﯾﮫ وش ﻧﺳوي‬
A. Chest physiotherapy
B. Mechanical ventilation
C. Reducing oxygen requirements
D. Use of anincentivespirometer✅ ‫اﻟﺷﻲ اﻟﻲ زي اﻟﺷﯾﺷﮫ‬

7.A patient treated an insect bite with traditional alternative therapy of row garlic juice caused
strong skin reaction. The patient agreed not use it again after the nurse explained its harmful
effects. What should the nurse instruct him on discharge?‫ﺟـﺎﻟـس ﯾـﺎﺧـذ ﻋـﻼج ﻣـن ﻧـﻔﺳﮫ وش اوﺟـﮭﮫ اﻗـوﻟـﮫ م‬
‫ﯾﺎﺧذ دوﯾﮫ ﻣن وﺣده‬

// 2996699650 ‫ ﻣﺣﻣد ﺳﺎﻣﻲ‬/‫أ‬

A. Refrain self treatment ✅


B. Medical treatment better than traditional
C. Avoid communication injury
D. Changedressingfrequently
Adminsteration:

8. Woman diagnosed with breast cancer and she will be performed mastectomy. She asked the
nurse to not inform anyone except her husband. Which of the following ethical principle?‫ﺗﻘول ﻻ‬
‫ﺗﻌﻠم اﺣد‬
A. Confidentiality ✅
B. Privacy
C. Autonomy
D. Justice

9. The patient is postoperative and sedated.‫ ﻣﻧوم ﺗﻧوﯾم ﻛﺎﻣل‬Which of the following signs
require immediate intervention?‫اي ﻣن اﻻﺟﺎﺑﺎت اﻟﺗﺎﻟﯾﮫ اﻧﺗﺑﮫ ﻟﮭﺎ اﻧو اﻟﻧﺎزو ﻗﺎﺳﺗرك م ﯾﻛون ﺷﻐﺎل‬
A. Urine bag is full
B. Homeric bag is full with serosanguineous drainage
C. Intermediate nasogastric suction is not connected ✅
D. Nasalcannula4Lwithouthumidification

10. Postoperative patient is complaining pain. The pain level is 8 from 10. The nurse is busy.
What's the task should the nurse delegate for assistant nurse?
A. Determine pain level
B. Assess pain while checking vitals
C. Give anelgesic
D. Teach Patient Breathing Exercise To Relieve Pain✅

11. A client with cold flu ‫ ﻧزﻟﺔ ﺑرد‬. He is suspected of having systemic lupus‫اﻟذﺋﺑﮫ اﻟﺣﻣرا اش اھم‬
‫ ﻋﻼﻣﺗﮭﺎ‬erythematous. The nurse monitors the client, knowing that which of the following is
one of the initial characteristic sign of systemic lupus erythematous?
A. Weight gain
B. Fever, malaise
C. Elevated red blood cell count
D. Rash On Extremities And Face ✅

12. Who will teach about car /road safety ?‫ﻣﯾن ﯾﺧﺎطب اﻟﻣرﯾض وﯾﻛﻠم ﻋن اﻟﺳﻔﺗﻲ‬
A. 1Doctor
B. Nurse✅
C. Social worker
D. Administrator

13. The patient has refused to take the medication, what the nurse will be response?
‫ م ﯾﺑﻐﻰ ﯾﺎﺧذ اﻟدوا ﻓﯾﮫ ﺳؤال ﯾﻘول ﻧﻔس ﻛذا ﯾﻛون اﻧﺗوﻣﻲ‬A.Give me a good reason for not taking
the medication

// 2996699650 ‫ ﻣﺣﻣد ﺳﺎﻣﻲ‬/‫أ‬

B.Tell me why you don't want to take the medication


C.Convinced me why you shouldn't take the medication
D.If you will not take the medication , you will sign a waiver✅ ‫ااوﻗﻌﮫ ﻣﺳؤوﻟﯾﮫ‬

14. The patient is experiencing burn 20% TBSA. Which of the following hormone affected?
A. Ranin
B. Cortisol ✅
C. Anti diuretic

15. Patient diagnosed with nephrotic syndrome. See lab result


Hgb 12. 5 g/dl What is the most common complication for nephrotic syndrome?‫واﺣد ﻋﻧده دﻣﺞ ﺑﺎﻟﻛدﻧﻲ‬
‫اش اھم اﻟﻣﺿﺎﻋﻔﺎت‬
A. UTI
B. Peritonitis ✅ ‫اﻟﺗﮭﺎب ف اﻟﻐﺷﺎء اﻟﻣﺣﯾط ﺑﺎﻟﺑطن‬
C. Bladder infection
D. Hypertensionencephalopathy

16.A 57 year-old woman presents with complaints of pain in the bones of her toes, foot and
swelling on knees . What type of musculoskeletal disorder is most likely?‫ھذي اﻋراض اﯾش‬
‫اھم ﺗﺣﻠﯾل ﻟﻧﻘرس ﯾرك اﺳد‬
A. Osteoarthritis.
B. Rheumatic arthritis
C. Osteomyelitis
D. Gout✅

17. The nurse has been set priorities for health problem. She is starting to management of time
board.‫ ﺗﺑﻐﻰ ﺗﺑداء ﺗﺣدد اﻟوﻗت ھل طوﯾل او ﻗﺻﯾر‬Which of the following step of nursing process?
A. Planning✅
B. Diagnosis
C. Intervention
D. Assessment

18. Woman came to outpatient for follow up. She complained from dizzness and fatigue. The
nurse refered her to relax inside empty room. She suddenly fell down on the floor then she
has faintaing and loss of conciousness. The nurse examined her and assessed vital signs.
Bl. P 90/50 mmhg , HR 120 b/m, RR 23 b/m.
What is the first intervention for her?
A. Notify the doctor
B. Elevate her leg ✅
C. Check Physical assessment and vital signs

19. Patient ABG PH 7.38, Paco2 50, Hco3 6 normal. What's the ABG interpretation?
A. Compensated Metabolic alkalosis

// 2996699650 ‫ ﻣﺣﻣد ﺳﺎﻣﻲ‬/‫أ‬

B. Uncompensated Metabolic acidosis


C. Compensated Respiratory acidosis ✅
D. UncompensatedRespiratoryalkalosis

20. Patient postoperative after hip surgery. What's the realistic goal should the nurse accomplish
?‫ﺑﻌد اﻟﻌﻣﻠﯾﮫ ﯾﻛون ﻋﻧدي ھدف ﻗﻠﯾل اﻟﻣدى‬
A. Pain control within one hour of medication Sitting three times per day ✅
B. Able to drive a car

21. The patient has exhausted, fatigue due to severe pain the level 6 /10, temp 37,5. What is the
most appropriate Short term goals?
A. Alleviate exhaustion
B. Manage pain and decrease temperature ✅

22. Patient complained from Loss of consciousness, The patient diagnosed with meningitis.
What's the first action for the patient
A. Neurological assessment ✅
B. Observe for seizures

23. The patient ate around 20% of meal. He can not complete food. He was experiencing Too
tired after eating. What should the nurse do?‫ﯾﺎﻛل وﺗﻌب م ﯾﻛﻣل اﻟوﺟﺑﮫ ﻧﻘوﻟﮫ ارﺗﺎح ﺷوي ﺑﻌدﯾﻧن ﻛﻣل‬
.Head done
Encourage fluids intakes
Offer food after rest ✅
Encourage him to take the protein part of diet

24.a nurse is caring for a patient who had a cerebrovascular accident 30 minutes ago with
residual right-side hemiparesis. The nurse places a trochanter roll extending from the patient's
iliac crest to the mid. What is the most likely complication?‫ﻋﻧده ﺟﻠطﮫ وﺿﻌف ف اﻟﺟﮭﮫ اﻟﯾﻣﯾن وھو ﻣن ﻣده‬
‫طوﯾﻠﮫ ﯾﺻﯾر ﻟو ﺷد ﻋظل ف اﻟﻔﺧذ‬
A. Adduction of the leg
B. External rotation of the hip
C. Muscle spasms in the thigh ✅
D. Flexioncontracturesoftheknee

25. Patient with expressive aphasia has impaired communication What is the short term goal for
him to increase verbalization?‫ﻋﻧدھﻣﺷﻛﻠﮫ ف اﻟﺗﻌﺑﯾر وش اﻟﮭدف ﻗﯾل اﻟﻣدى‬
A. Able to communicate effectively within a week
B. Anticipate all patients needs
C. Able to determine basic needs ✅

26. 44 years year old obese patient was subjected for a surgery called bariatric surgery, gastric
bypass to be specific. The nurse and the physician are giving information about the
procedure to the patient. The patient understands the procedure if he stated which among
the following?‫ﻋﻣﻠﯾﺔ اﻟﺗﻛﻣﯾم اش اﻟﮭدف ﻣﻧﮭﺎ ﻧﻘﻠل ﺣﺟﻣﮭﺎ‬

// 2996699650 ‫ ﻣﺣﻣد ﺳﺎﻣﻲ‬/‫أ‬

A. ''Same effect with liposuction''


B. ''An opening will be placed in my abdomen''
C. ''The surgery will reduced the size of my stomach'' ✅
D. ''Easiestwaytoloseweightnoneedtoexerciseaftersurgery''

27.72 year-old patient bedridden. He is complaining sever abdominal pain and suffering from
loss of skin integrity. What is the most appropriate Diagnosis?‫م ﯾﻘدر ﯾﻘوم ﻣن اﻟﺳرﯾر‬
A. Addison's crisis ✅ ‫ﻏده ﻓوق اﻟﻛﻠﯾﮫ ﻧﺷوف ﺗﺣﻠﯾل اﻟﻛرﺗزون ادا ﻛﺎن ﻧﺷﺎطﮭﺎ ﻋﺎﻟﻲ ادﺳون ﻗﻠﯾل ﻛﺎﺷﻧق اﻻدﺳون ﯾﻛون اﻟوزن‬
‫ﻧﺎزل واﻟم ف اﻟﺑطن‬
B. Thyrotoxicosis
C. Cushing syndrome

28.What is the impairment definition?


Social isolation
Difficulties in executing activities
Difficulties in body functioning such as extremity loss ✅

29.72 year-old patient bedridden. His neighbor found that patient has loss of consciousness and
he brought him to hospital. The nurse provide long term nursing care plan for patient. What is
the most important consideration?‫م ﯾﻘدر ﯾﻘوم واﻟوﻋﻲ ﻗﻠﯾل اش اﻟﮭدف طوﯾل اﻟﻣدى‬
A. Previous injuries on his body ✅ ‫ﻧﺷوف اﻟﺳﻔﺗﻲ ﻟو طﺎﯾﺢ‬
B. Identify Other medical illness
C. Bedridden status

30 .Patient with ABG ph low, pco2 high, hco3 normal what is expected signs and symptoms for
the patient ?
A. Vomiting and difficult breathing SOB ✅
B. Headache and vomiting
C. Shallow, rapid breath and vomiting
D. Kaussmalrespiration

31.The patient come to ER with head injury after 1 hr the expected Vital/Signs for patient is?‫ﺿرﺑﮫ‬
‫ف اﻟراس‬
A. Bp 126/85 RR 12 HR 78
B. Bp158/92 RR 9 HR 60✅

32.42 year-old patient went to the clinic for an eye consult . Patient complained of blurred
vision‫اﻟرؤﯾﮫ ﻣﺷوﺷﮫ‬
, ocular pain and head active . During assessment tonometry was done ( see results ) . Test
Result Normal Values Intraocular pressure 34 10-20mmHg . Which surgical procedure is the
most appropriate ?
a. Laminectomy
b. Laser trabeculoplasty ✅ ‫ﻟﯾزر ﯾﺳﺗﺧدم ﻟﻠﺿﻐط اﻟدﻣﺎغ ﻋﺎﻟﻲ‬
c. Incision and drainage
d. Extra capsular cataract extraction

// 2996699650 ‫ ﻣﺣﻣد ﺳﺎﻣﻲ‬/‫أ‬

33.A hospitalized 72-years-old man who uses a walker is received medication and must use the
bathroom several times each night. To promote the safety of the patient, which of the following
appropriate nursing action?‫ﯾﺗﺎج ﻣﺳﺎﻋده ف اﻟﻣﺷﻲ وھو ﯾروح ف اﻟﻠﯾل وﯾرﺟﻊ اﻟﺣﻣﺎم اﺧﺎف ﻋﻠﯾﮫ اﻧو ﯾطﯾﺢ‬
A. Keep the side rails up
B. Leave the bathroom light on
C. Provide a bedside commode ✅ ‫ﯾﺟﻠﺳو ع ﻛرﺳﻲ ﻣﻔﺗوح ﻣن ﺗﺣت‬
D. Withholdthepatient‘sdiureticmedication

34.70 years- old woman presents with increasing dyspnea on exertion. She feels breathless and
restless while performing household tasks, such as making the bed and sweeping the floor. Her
previous medical history includes a myocardial infarction at 57 years old. She sleeps with her
head elevated on three pillows. Examination reveals bilateral basal crackles and cold, damp
skin.‫ﻛﺎﻧت اﻟﻣﺷﻛﻠﮫ ف اﻟﺟﮭﮫ اﻟﯾﻣﯾن ﻣن اﻟﻘﻠب ﯾﻛون اﯾدﯾﻣﺎ ف اﻟﯾدﯾن واﻟرﺟول واذاﻛﺎﻧت ﯾﺳﺎر ﺗﻛون اﻻدﯾﻣﺎ ف اﻟرﺋﮫ اش ﯾدل ع اﻻدﯾﻣﺎ‬
‫ف اﻟرﺋﮫ ﻋﻧده ﺧﺷﺧﺷﮫ ف اﻟرﺋﮫ‬
Blood pressure 172/94 mmhg Heart rate 94/min Respiratory rate 36/min Temperature 37.1 C
Oxygen saturation 90% on room air
Which heart chamber most likely failed first?
a- Right atrial
b- Rightventricle
c- Left ventricle
✅ d- Leftatrial

35. a patient is being admitted to the Recovery Room following a thyroidectomy. The back of the
neck wound is covered with dressing during the first 15 minutes, the patient started working
and having diarrhea. A general assessment is performed with special attention given for the
high risk for hemorrhage where would bleeding most likely occur?
A. Stool
B. Vomitus
C. Dressing
D. Back of neck

36. patient is being weaned off from the mechanical ventilator is about to hook the endotracheal
tube to oxygen at FiO2 of 40 Which of the following oxygen administration device is the best
the nurse in this situation?
A. Ambo bag
B. Ventura mask
C. Tracheostomy collar
D. T-piece/ Briggs

37. Detailed history and physical examination Patient‘s acceptance for the treatment plan
Family‘s involvement and consent for her treatment . is the name of procedure?
Myringoplasty
Tympanoplasty ‫✅ ﻋﻣﻠﯾﮫ ﺗﺟﻣﯾل ﻟﺗﻣﺑﻧك ﻣﻧﺑرﯾن ﻻﻧو م ﯾﺳﻣﻊ ﻧرﻛب ﺳﻣﺎﻋﮫ داﺧﻠﯾﮫ‬
Tympanotomy

// 2996699650 ‫ ﻣﺣﻣد ﺳﺎﻣﻲ‬/‫أ‬

38.Patient has AV fistula in his leg. The patient came to dialysis unit for session. When the
nurse check the fistula, She found that fistula not working. What is the first action?‫ﻓﯾﮫ ﺳؤال‬
‫ﻋﺷﺎن اﻋرف اﻧﮭﺎ ﺗﺷﺗﻐل اﻟﻣﺳﮭﺎ ﺑﺎﻟﯾد اﺳﻣﻊ ﺻوت اﻟدم‬
A. Auscultate the fistula
B. Notify the physician✅
C. Monitor vital signs

39) Which of the following is most appropriate to delegate assistant?


A. Insertion of an oral airway
B. Changing postoperative dressing
C. Initial interview on a newly admitted patient
D. Repositioning a patient to side-lying position

40. A 15-year-old girl is admitted to the hospital with diarrhea. She has been repeatedly
vomiting for now weak and lethargic. She is oriented to time to questions appropriately.
The nurse prepares temperature using an electronic thermometer. Which measurement
would be most appropriate?
A. Oral
B. Rectal
C. Axillary
D. Tympanic

41. A 35-year-old patient was admitted to the Emergency Department accompanied by her
include severe epigastric pain, abdominal tender the last 24 hours. She was anorexic and
had since the last few hours. She was feeling elimination and body fluid loss. Which of the
following should the nurse do to exertion
A. Explain that she will feel better soon
B. Show support and understanding
C. Encourage her to drink fluids
D. Help he rest undisturbed

42. A patient complains of pain when standing upright hump on the upper back. In the past year
slightly shorter. The doctor has suggested tests to what is the best atppropriate intervention?
‫اش اﻻﺧﺗﺑﺎر اﻟﻠﻲ ﺑﺳوﯾﮫ اﺷﻌﮫ طﺑﻘﯾﮫ‬
A. Instruct the patient in the use of prescribed magnesium supplements
B. Have the patient sleep propped on two pain
C. Prepare the patient for a CT scan of both
D. Instruct the patient in the use of vitamins

43. A 56-year-old patient was diagnosed with instructions about administration of insulin
Regular Insulin and Neutral Protamine Hagedo subcutaneously every morning? Which of
the following is right period for (in minutes) after administration?

A. 10-15
B. 30-60
C. 60-90

// 2996699650 ‫ ﻣﺣﻣد ﺳﺎﻣﻲ‬/‫أ‬

D. 240-360

44. In a post-operative patient in the Surgical Unit most indicative of a developing complication?
A. Increasing alertness
B. Weak and rapid pulse
C. Negative Homans' sign
D. Minimal bowel sounds in four quadrants

45. A 19 year-old girl was scheduled for the extraction of her w under general anesthesia. Her
preoperative anesthesia was done and the consent was signed by her, but she refused her
nail polish and trim her long nails, which was requirement. What should be the nurse‘s action
in response to her refusal?
A. Explain why nails need to be cleaned
B. Respect the patient‘s right to refuse
C. Record and inform physician
D. Remove as per protocol

46. A 60-year-old man client had a permanent complains of chest


pain and dyspnea with rapid feels suffocated and appears
restless

Blood pressure 100/70 mmHg


Heart rate 96/min
Respiratory rate 32/min
Temperature 37.2C
What is the immediate nursing intervention?
A. Monitor and report findings of chest
B. Chest X-ray to identify dislocation of
C. Manage pain with medication as ordered
D. Administer oxygen as ordered

47. Mr. A attended in outpatient clinic with symptoms of Shortness of breath, diarrhea and
severe respiratory distress Which of the following is the best diagnosis of Mrs. A case?
A. Corona virus
B. Swine Flue
C. Zika virus
D. Hepatitis

48. which of the following patients the nurse should see first?
A. Patient complaining of muscle aches and fever
B. Patient scheduled for electrocardiography
C. Patient newly diagnosed with hypertension
D. Patient with complaint of chest pain

// 2996699650 ‫ ﻣﺣﻣد ﺳﺎﻣﻲ‬/‫أ‬

49. A 43-year-old women is evaluated for breast cancer during a routine screening procedure.
During the physical examination, the nurse palpates a lump in the right upper outer quadrant
of the breast. The patient requests that the nurse give the information only to her spouse.
Which ethical principle is the nurse up holding?
A-utonomy
B-Justice
C-Confidentiality
D-Beneficence

50. A client is receiving supplemental oxygen and the nurse is monitoring the client oxygen
saturation level Using pulse oximetry the nurse notify the physician if the client reaches
Which oxygen saturation level?
A-98%
B-92%
C-94%
D-96%

51. A conscious victim of a motor vehicle accident arrives at the emergency department. The
patient is Gasping for air, is extremely anxious, and has a deviated trachea. What
diagnosis should the nurse? Anticipate?
A-pleural effusion.
B-tension pneumothorax
C-pneumothorax
D-cardiac tamponed

52. A nurse calls together an interdisciplinary team with members from medicine, social service,
the clergy, and nutritional services to care for a patient with a terminal illness. Which of the
following types of care would the team most likely be providing?
A- Palliative
B-curative
C-respite
D-preventive

53. A 66 old women presents to the emergency department triage unit with labored breathing.
She appeared anxious and is sweating profusely across the forehead. The symptoms had
begun Suddenly one hour before. The jugular veins ‫ ﺳﺑب اﻧﺗﻔﺎﺧﮫ ﺿﻌف اﻟﺟﮭﮫ اﻟﯾﻣﯾن ف اﻟﻘﻠب‬appear
distended and she has a productive cough. On Auscultation the respiration is wet and not
clear with coughing. Blood pressure 148/88 mmHg
Heart rate 94/min
Respiratory rate 26/min
Temperature 37.0C
Which triage acuity rating is most appropriate?
A-Urgent
B-Emergent

// 2996699650 ‫ ﻣﺣﻣد ﺳﺎﻣﻲ‬/‫أ‬

C-Non-urgent
D-Resuscitation

54-A nurse is documenting in patient records several events that occurred during home health
visits.Which of the following is an example of the correct way to document patient information?
A-Patient fell walking to bathroom. Busy preparing for sterile dressing change, when the patient
left the Room
B-Patient got out of bed without assistance. Denise any symptoms
C-Patient sitting in chair. Strict bed rest orders ignored
D-Patient showering. Electronic epidural infusion pump on floor by shower stall

55. The physician has written an order for the nurse‘s assignee have a 24-hour urine collection
sent to the laboratory for special. Which of the following should the nurse realize prior to
urine?
A. Start the urine collection at either 12:01 a.m. or 12:01pm
B. Provide enough sterile receptacles for the urine collection
C. At the start of the collection period, have the patient discard this urine o
D. Inform the patient that they must save all urine for 24 beginning at 12:01 a.m.

56. After teaching the deep breathing and coughing who is undergoing a surgery, the nurse
asked demonstration and then helped him in correcting, What part of therapeutic
communication is used?
A. Evaluation
B. Intervention
C. Identification
D. Demonstration

57) Which of the following represents secondary‫ ﻋﻘم ﺛﻧوي ﯾﺟﻲ ﺑﻌد ﺣﻣل واﻻوﻟﻲ م ﻗت ﺣﻣﻠت‬infertility?
A. Male infertility
B. A couple that does not conceive
C. Infertility that occurs after previous pregnancy
D. Infertility lasts for more than 3 years

58. Which of the following is most appropriate to delegate assistance


A. Insertion of an oral airway
B. Changing postoperative dressing
C. Initial interview on a newly admitted patient
D.Repositioning a patient to side-lying
position

// 2996699650 ‫ ﻣﺣﻣد ﺳﺎﻣﻲ‬/‫أ‬

59) A nurse is leading an educational session on the correct use of oral contraceptives. One of
the attendees ask the nurse what to do if she missed taking an oral contraceptive for one day?
Which of the following should be the nurse advice to her?‫ﺗـﺎﺧـذ اﺣـﺑوب ﻣـﻧﻊ اﻟﺣـﻣل وﻧﺳـﯾﺗﮭﺎ ف ﯾـوم ﺗـﺎﺧـرت ﻋـن‬
‫وﻗﺗﮭﺎ وﺗذذﻛرت ﺗﺎﺧذھﺎ ع طول وﺑﻌدﯾن ﺗرﺟﻊ ﻟوﻗﺗﮭﺎ اﻻﺻﻠﻲ‬

A. Continue as usual with no back up contraception


B. Take an active pill immediately and take the next pill at The usual time
C. Take two pills as soon as possible and then one pill daily At the usual time
D. Use back up contraception such as a condom for the Next 7 days

60. An elderly patient who has an aortic aneurysm Intensive Care Unit to a Medical Surgical
Unit on day. While assessing the client, a nurse notes extremities and is unable to palpate
the pedal pulse ‫?ﺿﻌﯾف اﻟﻘدم ﻧﺑض‬which intervention should the nurse implement
A. Wrap the lower extremities with warm
B. Use a Doppler ultrasound to reassess the
C. Elevate the extremities above heart level
D. Place a bed cradle over the bed to levitate

61. During surgery requiring general anesthesia and a carotid pulse is not palpate. How many
compressions per minute should be
A. 50
B. 60
C. 80
D. 100

62. A 40-year-old man. Smoker, presents to the clinic on examination, the toes are cold to the
touch extremities is pale to blue. The pedal pulse. Examination of the fingers shows small
ulceration the skin. Blood glucose testing is normal and then history of diabetes on
admission

Blood pressure 140/90 mmHg


Heart rate 86/min
Respiratory 22/min
Oxygen saturation 98% room air

Which of the following would be most effective?


A. Antibiotic administration
B. Reduced fat intake
C. Smoking cessation
D. Regular exercise

// 2996699650 ‫ ﻣﺣﻣد ﺳﺎﻣﻲ‬/‫أ‬

63) A man is to be discharged from the General appendectomy. The precautionary measures
plans are discussed with him what is the most important desired outcome after?
A. Remain free of post-surgical complications
B. Report fever, redness or drainage from
C. Use pain management techniques apropos
D. Resume gradual activities and avoid weight

64. A 59-year-old women is admitted in the Medical her arms and legs. The muscular strength
progressively decreased within one year. She pattern and has difficulty in swallowing. Which
of the following nursing diagnosis require?
A. Risk of choking due to disturbed swallowing
B. Weakness and fatigue due to lower muscle
C. Disturbed breathing due to chest muscle
D. Disturbed activities of daily living due to

65. A nurse diagnosis a patient with readiness for the spiritual, this diagnosis is classified within
which of the nursing diagnoses?
A. Acute nursing diagnoses
B. Risk nursing diagnoses
C. Wellness nursing diagnoses
D. Possible nursing diagnosis

67) Which of the following patients the nurse should see first?
A. Patient complaining of muscle aches and fever
B. Patient scheduled for electrocardiography
C. Patient newly diagnosed with hypertension
D. Patient with complaint of chest pain

68) A 40-year-old women patient with Parkinsonism‫رﻋﺎش رﻋﺷﮫ اھم ﻋﻼﻣﺎﺗﮫ اﻟﯾد ﺗﮭﺗز ﺑﺳﺑب اﻟدوﺑﺎﻣﯾن ﻗﻠﯾل‬
‫ ﻧﻌطﯾﮫ ﻋﻼج ﺷﺑﮫ ﺻﻠب‬Medical Ward. The patient stated that she has the past two weeks. The
nurse was planning to which type of diet is most suitable for Parkinsonism?
A. Solid
B. Liquid
C. Semi solid
D. Clear liquid

69. A post-surgery client has a normal assessment with 37.6C at 0800 hours later in the day,
client is he continues to be stable Based on client's current temperature reading actions
should be taken by a nurse?
A. Inform surgeon since discharge should be
B. Record temperature twice daily, at morning
C. Aspirin 81 mg should be administered
D. Teach client to use a spirometer and drink recheck temperature in two hours

// 2996699650 ‫ ﻣﺣﻣد ﺳﺎﻣﻲ‬/‫أ‬

70. A nurse is caring for patient who is being admitted tract infection. The patient feels cold and
shivering

Blood pressure 110/70 mmHg


Heart rate 110 /min
Respiratory 22 /min
Temperature 39.7C

Which of the following is the best nursing action?


A. Provide a hot drink
B. Cover the patient with light blanket
C. Start the air conditioning system
D. Turn off lights and close curtains

71. A nurse is assigned to care for a Muslim female emergency cesarean section. The patient
asks to wear a head scarf during the surgery. Which of the following is the most
appropriate?
A. Ask the patient to remove the scarf
B. Tell the patient that it is ok to wear the scarf
C. Ask the patient to wear a surgical cap
D. Tell the patient that a request will be during the surgery

72. A 62-year-old women admitted to the emergency department for the fourth time this year,
each time the patient comes with sever injuries and bruises in the body. What is your
responsibility as a nurse to prevent such incident to happen again?
A. Reports the assault to the local police and write a report
B. Provides information about safe shelter and support
C. Instructs the women to move away from her home
D. Discharge the patient to a safe shelter

73. A 46-year-old patient is admitted in the Female back pain which is graded 6 on the scale
of10 with the slight elevation in her blood pressure. The eat lunch which is a low sodium
diet documented that patient is uncooperative and lunch. What can be assessed by the
nursing document patient?
A. Subjective judgement of patient's statement
B. Misunderstanding of patient's attitude
C. Understatement of communication
D. Unethical evaluation of reality

// 2996699650 ‫ ﻣﺣﻣد ﺳﺎﻣﻲ‬/‫أ‬

74) A nurse is caring for a 72-year-old man patient is unsteady. The patient requests the nurse
to help nurse asked the patient to wait few minutes device to transfer him. Which of the
following transfer devices is the nurse to use?
A. Belt
B. Board
C. Handle
D .Mechanicallift

75. How can the nurse give health education to prevent or reduce

sinusitis‫ اﻟﺗﮭﺎب اﻟﺟﯾوب اﻻﻧﻔﯾﮫ‬by Health education?

A. Increase carbohydrate intake

B. Increase fluids intake

C. Avoid travel with air plane

76. The nurse is assigned to care for several patients, who should be
assessed first by the nurse?
A. A diabetic patient who will be discharged
B. A patient with rheumatoid arthritis who has
C. An asthmatic patient who is due for‫رﺑو‬
D. A patient with surgical incision who required

77. Community health nurse was visiting people in certain area for observation and assessment.
She found their high risk for lead poison. She refer all to get lab investigation and doing tests.
What is the type for that?

. A: Screening ✅ ✅ ‫ﺗﺣري‬
B:Surveillance c. Assessment
78.Patient with unstable angina. The chest pain is ruled out. What is the priority

nursing diagnosis?

A-Anxiety from threatened disease B. Pain related to Angina pectoris

79. The nurse was taking history from the patient and she asked one question

Then repeated that question again to patient. The patient said (Are you hearing me I answered
before). What should the nurse do to clarify?

A. Ask new question B. Use eye contact during communication

C. Ignore the patient

D. Repeat again the previous question

// 2996699650 ‫ ﻣﺣﻣد ﺳﺎﻣﻲ‬/‫أ‬

80. year-old man is being discharged from the post-operative Care Unit following a ransurethral
resection of the prostate. The nurse provides discharge information regarding the care of the
bladder catheter. Which method would be most effective in bladder retraining for this patient?

A. Scheduled urination every 2-3 hours ✅ B. Limit fluid intake before sleeping time

C. Perform pelvic floor exercises daily D. Increase fluid intake during the daytime

81. Quality team in meeting to set the Best measure to decrease liability for

fall...What is the best action for that?

A. Apply fall risk band around wrist B. Lower bed level C. Round observations on

patient D. Apply fall risk sticker on patient door ✅

82. A 32-year-old woman being hospitalized since six days for obesity treatment

she reported to the nurse ―I feel burning in my left calf for the last two days

―after nursing assessment the nurse found that a left thigh edema, local

tenderness, thigh circumference is more by 8 cm than right thigh and it is warmer

to touch, no trauma or wounds. What is the most possible cause for that?

A. Obesity B. Local cellulitis C. Hypertension D. Deep vein thrombosis ✅

83. Which patient is contraindicated from MRI?

A. Patient who has dye contract allergy B Patient who have cardiac pacemaker ✅

84. Woman came to ER with severe Vomiting. Vomiting not stop and the patient
was anxious and crying. She said I will die. What is the most appropriate

nursing diagnosis? A. Fear to unknown disease ✅ B. Distribution body image

85. How can the nurse give health education to prevent or reduce sinusitis by Health education?

// 2996699650 ‫ ﻣﺣﻣد ﺳﺎﻣﻲ‬/‫أ‬


A. Increase carbohydrate intake

B. Increase fluids intake

C. Avoid travel with air plane

86. year-old women patient with Parkinsonism Medical Ward. The patient stated
weeks. The nurse was planning to Which type of diet is most suitable for parkin?

A. Solid B. Liquid C. Semi solid ✅ ✅ D. Clear liquid

87. A 45-year-old man is admitted to the neurosurgery ward for the surgical

elevation of depressed skull fracture. He has episodic severe headache

with

seizure and is unable to concentrate. Which of the following initial nursing problem needs more
attention?

A. Disturbed coping and anger spells

B. Risk of injury to seizure

C. Disturbed communication and irritability

D. Pain management and comfort measure

88. Nurse manager tells one of the staff, ―I don‘t have time to discuss the matter

with you now. See me in my office laterǁ when the latter asks if they can talk

about an issue. Which of the following conflict resolution strategies did she use?

A. Smoothing B. Compromise C. Avoidance✅ D. Restriction

89. Patient with unstable angina. The chest pain is ruled out. What is the priority

nursing diagnosis?

A. Anxiety from threatened disease B. Pain related to Angina pectoris✅

// 2996699650 ‫ ﻣﺣﻣد ﺳﺎﻣﻲ‬/‫أ‬

90. which of the following statements regarding SLE(of systemic lupus

erythematous ) nurse should provide to group of people?

A- hypertension must be anticipated ✅ B- hypertension is not common with SLE.


C- the symptom will exacerbate more in winter and spring.

91. What is the most common hazard in hospitals‫? ﻣﺧﺎطر اﻟﻣﺳﺗﺷﻔﻰ‬

A. Chemical B. Physical✅ C. Biological D. Psychological

92. A 40 years old complains of severe pain, scheduled for surgery of

abdomen, medical history he smokes one packed of cigarettes per

day. Which of the following complications he might develop?

A- atelectasis‫ ✅ اﻧﺧﻣﺎص‬B- DVT C- constipation

93. What is the Advantage of Nursing documentation system??

a. Cost for hospitals

b. Spent time with patient.

c. Decrease of nurse‘s satisfaction

D. Decrease of documentation quality:

94. woman patient admitted with indwelling Foley catheter with a closed drainage

system was order collection for sterile urine specimen. Which of the following

steps is considered the best way to collect the sterile urine specimen?

A-Obtain the specimen from the drainage bag

B-Obtain the specimen from the aspiration port✅

C-Obtain the specimen from the first voiding in the morning

D- obtain a clean mid-stream catch cleaning the perineum

// 2996699650 ‫ ﻣﺣﻣد ﺳﺎﻣﻲ‬/‫أ‬


95. The patient has joint arthroplasty surgery‫ ﻣﻔﺻل ﺻﻧﺎﻋﻲ‬. The patient with urinary
catheter An d IV line is attached. He was on oxygen, what needs immediate
intervention?
A. Pain, redness, and swelling B. Shortness of breath and coughing✅
Answer: B

96. Pt when entered ER he said, my heart will get out of my chest I fell that I will
die he diagnosed with panic attack what is the medical problem that will be
developed if panic not controlled?
A) Respiratory acidosis
B) Respiratory alkalosis
C) Metabolic acidosis
D) Metabolic alkalosis
Answer: B

97. Patient will start TPN‫ ﺗﻐذﯾﮫ ورﯾدﯾﮫ‬and should started gradually to avoid which‫ﻓﯾﮫ ﺳؤال ﺗﺣت‬
‫ ﻣﻌدل‬:
A-hyperinsulinemia B-hyperinsulinemia C-hypoglycemia D-hyperglycemia Answer

98.He discusses the goal of the department. Which of the following statements is goal?
A. Increase the patient satisfaction rate.
B. Eliminate the incidence of delayed administration of medications.
C. Establish rapport with patients.
D. Reduce response time to two minutes.
Answer: A

99.After a code blue, a doctor announced the death of a patient. A signature


other begins to show signs of decreased level of consciousness. What is the priority
intervention of the nurse at this time?
A. Offer a meal
B. Ensure safety ✅
C. Set up an IV line
D. Offer a glass of water
Answer: B

100. A 21-year-old in oversized clothing presents to the hospital with of felling dizzy
And faint. The hair and nail appear thin and dry. The skin appears pale, and she has
Sunken eye sockets and tenting skin. Her body mass index is 16. She often induces
Vomiting after eating blood is collected for analysis (see lab results). *Blood pressure
90/52mmHg, Heart rate 118 /min, Respiratory rate 26/min, * Temperature 37 .2 C,
Oxygen saturation 97%.
A- ABG Test Result Normal value
B- HCO₃ 31 22 – 88mmo?l/L
C- PCO₂ 10.3 4.7-6.0 kPa
D- PH 7.50 7.36-7.45

// 2996699650 ‫ ﻣﺣﻣد ﺳﺎﻣﻲ‬/‫أ‬

101. Which nursing problems stem is the most appropriate? A. Impaired nutrition B.
Decreased cardiac output. C. Infective airway clearance D. Ineffective breathing
pattern
Answer: B

102. Pt when entered ER he said, my heart will get out of my chest I fell that I will
die he diagnosed with panic attack what is the medical problem that will be developed if
panic not controlled?
A) Respiratory acidosis
B) Respiratory alkalosis
C) Metabolic acidosis
D) Metabolic alkalosis
Answer: B

103.A nurse who works in the surgical unit at one of the hospitals was asked byThe
home health care nurse to make a home visit to a patient with colostomy, who had
been discharged the previous day in order to give him a follow-up care and
education which of the following nurses should do the assigned task?
A. Critical care nurse B. Psychiatric nurse C. Surgical nurse D. Community nurse
Answer: D

104.A 65-year-old women visited the gynecological outpatient history reveals that
she had 3 pregnancies, one abortion gestational age, had 2 normal deliveries. She
smokes 20 Her complaint is that she wets herself when she coughs embarrassing
for her? Which of the following can be considered as risk factors pelvic floor muscles?
A. Chronic coughing
B. Diabetes mellitus
C. Excessive spot
D. Sedentary lifestyle

105. Epidural anesthesia site?


A. L3 - L4
B. L1 - L2
C. T3 - T4
D. C1 - C2

// 2996699650 ‫ ﻣﺣﻣد ﺳﺎﻣﻲ‬/‫أ‬

106. Why do we use ―log rolling" Technique?


A) Facility good circulation
B) Avoid movement
C) To protect the spinal cord from moving

107.one-week short-term goal for a total hip replacement patient?


A sits up by himself three times a day.
B. state pain control

108.CSF contains NOT all of the following except:


A. Insulin. ✅
B. RBC.
C. WBC.
D. Protein.

109. Why should the nurse while doing CPR ensure from complete chest recoil?
A - Allow the lung to expand ✅
B- increase cardiac output

110. What is the best nutrition teaching instruction for patient with Parkinson‫رﻋﺷﮫ‬
disease? A- Choking
B-Drooling ✅ ✅ ‫ﺳﯾﻼن اﻟﻠﻌﺎب ﯾﺑﻠﻌﮫ‬
C- Aspirations
D- Dysphasia

111. Which of the following is the primary level of prevention?


A. Detect and treat existing disease.
B. Prevent illness or injury occurring.
C. Reduce the extent and severity of health problem.
D. Minimize disability and restore to prevent function.

112. A nurse is caring for a patient receiving total parenteral nutrition. (TPN). The
patient reports the sudden onset of feeling short of breath and anxious. The nurse hears
crackles in bilateral lower. lobes of the lungs and the patient‘s O2 saturation is 90%on
room air. The nurse must IMMEDIATELY:
a. Turn off the TPN.
b. Notify the physician.
c. Asses the patient‘s capillary blood glucose level
d. Attempt to suction the patient‘s airway

// 2996699650 ‫ ﻣﺣﻣد ﺳﺎﻣﻲ‬/‫أ‬

113.Which of the following nursing diagnoses has the highest priority when caring
for an older adult client with Alzheimer's disease?
a) Impaired physical mobility
b) Impaired memory
c) Self-care deficit
d) Risk for injury

114. Bedridden PT nurse is doing pathing for him and she used lotion after bathing
to prevent pressure ulcer for patient. What is appropriate action??
A. Dry patient after bathing
B. Use Air mattress
C. Keep the sheet loosely.

115. The nurse was doing research with two groups of smokers, the first group took
Nicotine Patch and the second group not achieved the target goal, what is the research
type?
A. Case Study
B. Cohort study

116. which of the following statements regarding SLE nurse should provide to
group of people?
A- hypertension must be anticipated
B-hypertension is not common with SLE.
C- the symptom will exacerbate more in winter and spring.

117. 32 year-old woman patient was brought to the Outpatient for the removal of
stitches on her left cheek which was treated nine days back after being involved in road
traffic accident. She covers her face completely and requests to be seen by a female
doctor. The site of the wound was red, swollen and some pussy points were visible. She
states that she did not wash her face since her accident and kept her face covered all
the time as she did not want anyone to see it. What is the most appropriate nursing
diagnosis?‫ﻋﻧده ﺧﯾﺎطﮫ ف اﻟوﺟﮫ وﻛﺎﻧت ﻣﻐطﯾﮫ وﺟﮭﮭﺎ وﻟﻣن ﻓﻛت طل ﻋﻧدھﺎ اﺣﻣرار وﺻدﯾد‬
A. Hopelessness
B. Social isolation
C. Anxiety
D. Powerlessness

// 2996699650 ‫ ﻣﺣﻣد ﺳﺎﻣﻲ‬/‫أ‬

118. Nurse manger float RN nurse from surgical ward to CCU which patient should
be assigned to this nurse?
A. Patient just transferred after coronary angiography.
B. Patient postoperative after open heart surgery
C. Patient on I. V Lasix have congestive heart failure
D. Patient had ventricular tachycardia and connected to cardiac monitor.

119. Nurse mangers pull out (float) nurse from medical ward to CCU which of the
Following Patients should be assigned to the nurse:
A. Patient just transferred from cardiac catheterization
B. Patient just received with unstable angina on heparin infusion.
C. Patient need discharge education about coronary stenting.
D. Patient on Lasix I. V have acute left ventricular failure.

120. A nurse is a member of the Nursing Research Council of the hospital. His first
assignment is to determine the level of patient satisfaction on the care they received
from the hospital. He plans to include all adult patients admitted from April to May, with
average length of stay of 3-4 days, first admission, and with no complications. Which of
the following is an extraneous variable of the study?
A. Date of admission
B. Length of stay
C. Age of patients
D. Absence of complications

121. There is crisis in hospital. Which of the following category of patients should
be evacuated first during fire???
A. Infant
B. Ambulatory‫اﻟﻠﻲ ﯾﻣﺷون‬
C. elderly
D. Critically ill

122. The nurse receives ECG for patient via Internet. What is consider?
A. Telehealth
B. Telecommunication
C. Telemedicine

123. The patient is postoperative cardiac surgery. The pulse rate is changed to
bradycardia. What is the most common cause of decrease Heart rate after operation?
A. Pain
B. Stimulate Vegas nerve
C. Anxiety

// 2996699650 ‫ ﻣﺣﻣد ﺳﺎﻣﻲ‬/‫أ‬

124. 35 years old female patient after taking breast biopsy she diagnosed with 3rd
stage breast Cancer She asked what is the purpose of chemotherapy?
A-it make no need for surgery.
B-destroy cancer cells and prevent it from spreading.
C- prevent recurrence of cancer.

125. which of the following the role of informatic nurse?‫اﻟﻠﻲ ﯾﺷﺗﻐل ف ھذا اﻟﻣﺟﺎل ﯾﻛون ﺑراﯾﻔﺳﻲ‬
A) software
B) design system

126. Why do we use ―log rolling" Technique?


A) Facility good circulation
B) Avoid movement
C) To protect the spinal cord from moving

127. 38-year- old patient is about to have lumber disk surgery. during preoperative
care, the nurse instructs the patient including the family. members how to do "log rolling"
to change patient position. One of the family members ask why they must do such action
in turning the patient. postoperative. Which of the following is the nurse best response?
A. Facility good circulation
B. Avoid movement
C. Prevent post-operative bed sore.
D. Makes changing of patient position easier.

128.A nurse is providing session regarding lung cancer, which statement indicates
teaching was not effective? ‫ﻋﻧده ﻛﺗﻠﮫ ﺧﺑﯾﺛﮫ ف اﻟﻠﻧق أي ﻣن اﻻﺟﺎﺑﺎت اﻟﺗﺎﻟﯾﮫ ﻟﯾﺳت ﻓﻌﺎﻟﮫ‬
A- I will eat fruit and vegetables
B- if I have COPD does not mean I will have lung cancer.
C- no need to worry because I do not smoke, my husband does.

129. Yemeni soldier caught in the war and he got many injuries. The patient anxious and
he is speaking about death for the nurse.What is the most appropriate action for the
nurse??
A -You say no or yes.
B. Why do you say you are dying?
C. Why you say you will die?
D. Restating for patient wards

130. Infant came to ER and during the examination, the doctor said he had spinal
bifida occult. What does the doctor know about this condition?‫ﺟﻧﯾن ﺟﺎﺑوه ع اﻻﺳﻌﺎف‬
‫وﺧﻼل اﻟﻔﺣص اﻟطﺑﯾب ﻗﺎل ﻓﯾﮭﺎ ﺗﺷوھﺎت ﺧﻠﻘﯾﮫ ف اﻟﺣﺑل اﻟﺷوﻛﻲ ﻣﻘﻔﻠﮫ اﻟﺳؤال ﻛﯾف ﻋرف اﻟدﻛﺗور‬
A. Infant has hair patch at back
B. Infant has open wound on back

// 2996699650 ‫ ﻣﺣﻣد ﺳﺎﻣﻲ‬/‫أ‬

131. The most risk of death /Mortality??


A) hemorrhage
B) pulmonary embolism
C)cervical cancer
D)Cardiac disease

132.A 48 year old patient in the male Surgical Ward had his gall bladder removed
through laparoscopic cholecystectomy24 ‫ اﺳﺗﺋﺻﺎل ﻣراره‬hours ago. While evaluating his
general condition, the patient appears lethargic and complains of severe nauseated
feeling along with discomforting the abdomen. What nursing problem needs to be
prioritized?‫اش اﻟﻣﺷﻛﻠﮫ وﻓق اﻻوﻟوﯾﺎت‬
A. Disturbed metabolism due to higher energy demand
B. Weak and lethargic due to low food and fluid intake
C. Nausea and vomiting due to slower gut movement ✅
D. Impaired comfort related to postsurgical effects

133. DM patient at night shift with cold skin, tachycardia, diaphoresis what you will
do fist?
‫ھذي ﻋﻼﻣﺎت اﻧﺧﻔﺎض اﻟﺳﻛر‬
A. check blood glucose.
B. give him cup of orange juice

134. A 19-year-old boy has been hospitalized with fracture in upper and lower
extremities after accident then provided with casts for upper and lower limbs. Which
of these Nursing diagnoses? should the nurse consider in the Nursing plan of care.
According to his age??
A. Impaired social interaction
B. Alteration in body image
C. Risk for infection
D. Anxiety

135. Woman came to outpatient for follow up . She complained of dizzness and
fatigue. The nurse referred her to relax inside empty room . She suddenly fell down on
the floor then she has fainting and loss of conciousness. The nurse examined her and
assessed vital signs . Bl . P 90/50
mmhg , HR 120 b / m , RR 23 b / m What is the first intervention for her ?‫ف ﺳؤال ﺣﻠﯾﻧﮫ ﻛﺎن‬
‫ﻣﺳوي ﺣﺎدث وﺿﻐطﮫ ﻧﺎزل وﻋﻧده ﺻدﻣﮫ ﻋﺻﺑﯾﮫ اذان ﺷﻔﺗﮫ اﻟﺻدﻣﮫ اﻟﻌﺻﺑﯾﮫ ﯾﻛون ھﺑوط ﻣﻔﺎﺟﺊ ف ﺿﻐط اﻟدم ﻻزم اﻓﮭم اﺧﻠﻲ‬
‫اﻟﻣرﯾض ﻋوﺿﻌﮫ م اﻏﯾر وﺿﻌﮫ ﻋﺷﺎن م ﯾﺻﺎب ﺑﺟﻠطﮫ‬
‫ھذي اﻟﺻدﻣﮫ اﻟﻌﺻﺑﯾﮫ ﯾﻛون اﻟﺿﻐط ﻧﺎزل واﻟﻧﺑض ﻧﺎزل ﺑﺎﻟﻌﺎده ف اﻟﺣوادث اﻟﺛﺎﻧﯾﮫ ﯾﻛون اﻟﺿﻐط ﻧﺎزل واﻟﻧﺑض‬
‫ﻋﺎﻟﻲ‬
A. Notify the doctor
B. Elevate her leg
C. Check Physical assessment and vital signs

136. 72 year-old patient bedridden. His neighbor found that patient has loss of
consciousness and he brought him to hospital. The nurse provides long term

‫أ‪ /‬ﻣﺣﻣد ﺳﺎﻣﻲ ‪// 2996699650‬‬


nursing care plan for patient. What is the most important consideration?
A. Previous injuries on his body‫ﻻﻧو ﻛﺎن طﺎﯾﺢ‬
B. Identify Other medical illness
C. Bedridden status

137. The patient scheduled for operation. The doctor informed the patient about
Expected complication and the outcomes May be not acceptable. What is the
nonpreferable Ethical principles?‫ﺑﺟﻲ داﯾم اش اﻟﻠﻲ ﻏﯾر ﻣﻘﺑول ﻣن اﻻﺟﺎﺑﺎت‬
A. Malpractice
B. Negligence
C. Ethical dilemma ✅ ‫ﻣﻌظﻠﮫ اﺧﻼﻗﯾﮫ‬
D. Molars concuss

138.Person work in factory. Every day expose to hazard. What type of hazard?
A— Chemical
B — Environmental
C— Biological
D— Psychological

139. The nurse is caring for a client who has had a chest tube inserted and
connected to water seal drainage. The nurse determines the drainage system is
functioning correctly when which of the following is observed?‫اش اﻟﺷﻲ اﻟﻠﻲ ﯾدل ع اﻧو‬
‫ﺷﻐﺎل اﻟﺗﯾوب‬
A— Continuous bubbling in the water seal chamber
B— Fluctuation in the water seal chamber
C— Suction tubing attached to a wall unit D—
Vesicular breath sounds throughout the lung fields

140. A Pt with congestive heart failure and severe peripheral edema has a nursing
diagnosis of fluid volume excess What are the two MOST important interventions
for the nurse to initiate?‫اﻟﺗدﺧل اﻻوﻟﻲ‬
A— Diuretic therapy and intake and output
B—Nutritionaleducationandlow-sodiumdiet
C— Daily weight sand intake and output
D— Low-sodium dietand elevatelegs whenin bed

141.Patient postoperative after hip surgery. What's the realistic goal should the
nurse accomplish ?
A. Pain control within one hour of medication‫ﻻﻧو ھدف ﻗﻠﯾل اﻟﻣدى‬
B. Sitting three times per day
C. Able to drive a car

‫أ‪ /‬ﻣﺣﻣد ﺳﺎﻣﻲ ‪// 2996699650‬‬


142. Postoperative patient is complaining about pain. The pain level is 8 from 10.
The nurse is busy. What's the task should the nurse delegate for assistant nurse ?
A. Determine pain level
B. Assess pain while checking vitals
C. Give analgesic
D-Teach patient breathing exercise to relieve pain

143. The hospital applied statistical study for Adult patients who admitted from
January to last February and They found MOST ofpatientthelengthofstayfrom3to4daysin
hospital. They discharged without any complication What is the Extraneous eliminated
Variable?
A, Age
B. Length of stay
C. Free of Complication
D. Date of admission

144.A nurse is educating a diabetic mother on symptoms of. Which of the following
symptoms stated by the mother indicate education was effective?
A. Polyuria
B. Flushing
C. Blurred vision rvmouth

145. Patient with COPD came to ER. The patient was very exhausted. He was
complaining shortness of breathing and cough. What is the most appropriate PaO2
percentage should be given for him?
A— 55 or below mmhg
B— 80 orhigh
C— 70 mmhg
D— 90 mmhg

146. Panic attack, respiratory alkalosis

147.During providing care to the patient. There was emergency situation in the ward.
She already given medication to patient but still no documentation yet. What should the
nurse do to prevent any error result from delayed documentation?
A. Document at the end of the shift
B. Write note that drug done and the dose✅
C. Don't go to emergency situation until you finish documentation

148.An 18-year-old client with acute lymphocytic leukemia is admitted to the bone
marrow transplantation unit. His family is having trouble dealing with the emotional
and financial pres-sures of his disease. The Pharmacist, nurse, doctor,

// 2996699650 ‫ ﻣﺣﻣد ﺳﺎﻣﻲ‬/‫أ‬

Physiotherapy meet to discuss the care plan for the patient. What is the type of
care interaction?
A- Casemanagement
B. Interdisciplinary
C. Cooperation
D. Collegiality

149.Nurse manger pullout(float) RN nurse from medical ward to CCU which of the
following Patients should be assigned to the nurse:
A- Patient just transferred from coronary artry surgery
B- B-Patient just received with unstable angina on heparin infusion
C. Patient need discharge education about coronary stenting
D.Patient on admission need assessment

150.Which of the following related to Low pitched breath sound??


A. Crackles
B. Rhonchi
C. .Wheezing
D. Vascular

151.Yemeni soldier caught in the war and he got many injuries. The patient anxious
and he is speaking about death for the nurse. Whatis the most appropriate action
for the nurse??
A .You say no or yes.
B. Why do you say you are dying?
C.Why you say you will die?
D. Restating for patient wards

152. nurse is caring for 6 patients. Providing for them all care and
activities. What is the type of nurse according to nursing care
deliverysystem?
A⁃ primary
B⁃ Case management
C⁃ Functional

153. Which of the following one of Complication of DVT (Deep Vein Thrombosis)
A. pulmonaryembolism
B. Thrombophlebitis

154. A nurses is providing teaching session for pre-operative patient before


appendectomy about how to use spirometer, which of the following nursing
process?

// 2996699650 ‫ ﻣﺣﻣد ﺳﺎﻣﻲ‬/‫أ‬

A. Assessment
B. . evaluation
C. .implementation
D. . diagnose

155. What is the updated technique for CPR According to AHA ?


A. 30:2120at5cm
B. 15:2120at5cm
C. . 30:2100at4cm

156. A nurse is planning to assess the corneal reflex on unconscious client. Which
of the following is the safest stimulus to touch the client's cornea ?
A. Cotton buds
B. Sterile glove
C. Sterile tongue depressor
D.Wisp of cotton

157. The patient scheduled for operation. The doctor informed the patient about
expected complication and the outcomes May be not acceptable. What is the
nonpreferable ethicalprinciples?
A. Malpractice
B. Negligence
C. Ethical dilemma
D. Molars concussi

.158- Patient developed ascites which procedure will help relieve this condition‫ﻋﻧد ﺗﺟﻣﻊ‬
‫ﺳؤاﯾل ف اﻟﺑطن ﻧﺳﺣﺑﮭﺎ‬:
A. Paracenthesis ✅ ‫اﺳم اﻟﻌﻣﻠﯾﮫ‬
B. Hemodialysis
C. Lumber puncture

159-30 year old man newly diagnosed with Diabetes. The patient has great knowledge
on how to monitor his blood glucose levels. But needs education on insulin
administration, dose, discard of medication. What should the nurse‘s outcome be?
A. ― the patient will show knowledge about insulin and insulin administration and
discard of needleǁ
B. ― the patient will return correct demonstration of aspirating the medication for
vial, administration of insulin and how, discarding needles in sharp box.ǁ

160-A nurse is caring for a patient receiving skeletal traction. Due to the patients severe
limits on mobility, the nurse has identified a risk for atelectasis or pneumonia. What
intervention should the nurse provide in order to prevent these complications?
A) Perform chest physiotherapy once per shift and as needed.
B) Teach the patient to perform deep breathing and coughing exercises. ✅
C) Administer prophylactic antibiotics as ordered.

// 2996699650 ‫ ﻣﺣﻣد ﺳﺎﻣﻲ‬/‫أ‬

D) Administer nebulized bronchodilators and corticosteroids as ordered

161. High reading of Blood Pressure what is the reason? ‫ﺳؤال ﻋﻛﺳﮫ ﺗﻛون اﻟﻛف واﺳﻊ واﻟﺿﻐط ﻧﺎزل‬
A. Small cuff ✅
B. Arm is elevated above heart level

162. What is complications after partial gastrectomy needs more teaching and ‫؟‬
reporting-to hospital
A. Fullness quickly
B. Gastric spasm ✅ ‫ﺗﺿﯾق ف اﻟﻣﻌده‬
C. Colic pain

163. What is symptoms of fluid exces.


A. High BP ✅
B. Low BP
C. Dizziness

164-.Which of following support nurse assumptions that pt risk for recurrence,


uncontrollable, lack of sleep‫وش اﺳوي ﻟو ﻋﺷﺎن ﯾﻧﺎم زﯾن‬
A-Pt avoid taking king makreal
B-Pt taking oatmeal for 2 years
C-Pt taking vitamin E for 2 year

165. Patient came to ER after motor accident. The patient has sever bleeding.
Postoperative head surgery the vital signs are Bl. P 90/60, HR 126 b/m
, respiration 24 b/m). The doctor ask the nurse to assess good tissue perfusion. Which of
What is Indication of Improve tissue perfusion?‫اش ﯾدل ع ﻟﻧو اﻟدم ﯾﻐذي اﻻﻧﺳﺟﮫ‬
A- decrease HR to 100
B - increase pulmonary wedge pressure
C -Increase Systolic BP to 86
D- 100 ml dark urine

166. The patient complained from stomachache continues after eating for 2 to 3 hours.
The patient has increased weight 3 kg during short time. Which of the following is the
most important nursing diagnosis?
A-Imbalance nutrition
B- Acute pain ✅
C-Fluid volume deficit

.167- A 48yearold patient in the male Surgical Ward had his gall
bladder removed through laparoscopic cholecystectomy 24 hours ago. While evaluating
his general condition, the patient appears lethargic and complains of severe nauseated
feeling along with discomfort in the

‫أ‪ /‬ﻣﺣﻣد ﺳﺎﻣﻲ ‪// 2996699650‬‬


abdomen. What nursing discharge instructions ?
A. return to hospital if there is no bowel movement for 1 week ✅
B. increase fluid intake
C. consume normal diet and restrict fiber

168. After left retinal‫ ﺷﺑﻛﯾﮫ‬detachment surgery‫ اﻧﻔﺎﺻل اﻟﺷﺑﻛﯾﮫ اش ﻧﺛﻘﻔﮫ‬. Which of the following
is the most appropriate health education ?
A. avoid bowel straining ✅ ‫ﯾﺗﺟﻧب ﺿﻐط اﻟﺑطن‬
B. talk with pt from right side

169.patient hospitalized then develop dysphagia and he has tablets, how to give him the
drug?
A. Dissolve it in water ✅
B. Divide the pill in half and give him one half
C. Open the capsule and place it under his tongue
D. Lookingforaliquidalternativetomedication?

170.patient with liver disease and take lactulose enema. what type of enema? A-feeding
B-cleaning
C-medicated ✅

171-. nurse take patient radial pulse half minute and found irregular what will the nurse
do?
A-check apical pulse
B-check radial pulse for full minute ✅
C-call the doctor

172-40 y old women she diagnosis with 4 cm fibroid mass what is the best interpreting
by the nurse ?‫اش اﻓﺿل ﺗوﺿﺢ ﻟﻠﻛﺗﻠﮫ‬
A. Carcinoma need hysterectomy ✅ ‫ﯾﺣﺗﺎج اﺳﺗﺋﺻﺎل‬
B. Carcinoma need chemotherapy
C. Benign disappear if no symptoms menopause

173. The nurse received the patient after hip surgery. The nurse is unable to locate an
older client's left popliteal pulse. Which action will the nurse take next?
1.Check for the femoral pulse.
2.Check for the pedal pulse.

3. Use doppler ultrasound for location . 4.Measure the blood pressure on the left thigh

174. The patient with gastroenteritis and he on I. V fluids N. S 500 ml starting at 13:00
pm. The doctor ordered the rate on infusion pump 125 ml/ hr. The nurse checked the
solution after 2 hours. She found empty bottle. The solution finished before timing. What
should the nurse do?
A. Check for Infusion pumps settings ✅

‫أ‪ /‬ﻣﺣﻣد ﺳﺎﻣﻲ ‪// 2996699650‬‬


B. Stop solution and start next at 17:00 pm
C. Administer new N. S now

175. During pre operative phase nursing intervention/ nursing role is ?


A. explain surgery to patient
B. obtain informed consent from patient
C. verify that patient and family understand informed
consent form ✅ ‫ﻧﮭم اھﻠﮫ ﻋن اﻟﻣواﻓﻘﮫ وﻧﺎﺧذ اﻟﻣواﻓﻘﮫ‬

176-A patient with server diverticulitis had surgery for placement of a colostomy. The
patient is upset, crying, and will not look at the colostomy. Which of the following would
be the highest priority nursing diagnosis at this time?‫ﻓﯾﮫ ﻓﺗﺣﮫ ف ﺑطﻧﮫ واذا ﺷﺎﻓﮭﺎ ﯾﺟﻠس ﯾﺑﻛﻲ‬
A. Knowledge deficit, colostomy care
B. Distorted body image ✅
C. Self-care deficit, toileting
D. Alteration in comfort

177-Which abnormal lung sound would require immediate intervention?


A. Stridor ✅ .‫زي اﻟﺻرﯾر‬
B. Crackles
C. Wheezing

.178- A 24 years old patient brought to emergency room after an accident and the nurse
noticed a leaking of clear fluid from the nose. A nurse anticipated the injury is :‫اﻟﻣﺎده اﻟﻧﺎزﻟﮫ‬
‫ ﻣن اﻻﻧف ھﯾﺎ ﻣن اﻟﻣﺎغ ﻣن‬csf ‫ﻟوﻧﮫ ﺷﻔﺎف ﻋﺷﺎن اﻟﺟﻠﻛوز‬
A. Basilar ✅
B. Frontal lobe
C. Temporary lobe
D. Neuropathic

179. Patient with thyroid storm after hyperthyroidism. He has high grade fever. What is
the additional expected symptoms? ‫ﯾﻛون ﻣﻧﺧﻔض اﻟﮭرﻣون واﻟﻌﻛس‬TcH‫اذا ﻛﺎن ﻧﺷﺎطﺗﮭﺎ ﻋﺎﻟﻲ‬
A-protroted eyeballs
B-increase sensitive to heat ✅

180. Patient came to ER with complaints of dark spots in eye. He diagnosed with retina
detachment. What should the nurse ask to patient that cause case?‫ﺳﺑب اﻧﻔﺎﺻل ﺷﺑﻛﯾﺔ اﻟﻌﯾن‬
A. do you have history of cataract ✅ ‫ﻣوﯾﮫ ﺑﯾﺿﺎ ف اﻟﻌﯾن‬
B. do you have hypertension

181. Which of the following sound gurgling breath sound at the end of expiration ?
a. Bronchovesicular
b. Adventitious breath ✅ l‫ﻣﺟﻣوﻋﺔ اﺻوات اﻟرﺋﮫ‬

‫أ‪ /‬ﻣﺣﻣد ﺳﺎﻣﻲ ‪// 2996699650‬‬


182. Patient with systemic Lepus arthritis complain pain in the joint She has butterfly rash
in her face what is most appropriate nursing action ?‫اش اﻟﺗدﺧل اﻟﻣدى‬
A. maintain skin integrity
B. B.manage pain discomfort relief

183. Postoperative patient with cataract procedure. What is the symptoms should the
nurse concern?
A-Blurred vision
✅ B-Slight
redness C-slight
discharge

184. Pt have sore throat, lesions in mouth that didn't cure , disphagia , loss of
weight. What's the expected diagnosis? A.AmGastritis
C. Tonsillitis
C.Laryngeal cancer
✅ D..Emphysema

185-A nurse I the medical surgical unital is review a plan of care elderly client with
chronic obstruction pulmonary disease ( COPD) limited mobility . the nurse notes that
the physical therapist change in the plan of care to progress ambulation from 50 to 100
times a day. Which action in necessary to ensure that the client need are?‫ﺑودﯾﮫ ﻟﻠﻌﻼج‬
‫اﻟطﺑﯾﻌﻲ ودﻛﺗور اﻟﻌﻼج ﻋدل اﻟﺧطﮫ اش اﺳوي اﺗﺻل ع دﻛﺗوره‬
A. Inform physical therapist of client respiratory status before progressing to ambulation
B. Instruct physical therapist not to proceed with ambulation nurse presence
C. Inform physician about physical decision to ambulation
D. D.Cancel referral to physical therapist

186. Which of the following lower postetior lung sound?


A.Bronchial B.Bronchovesicular C.Vesicular ✅
E. Both bronchial and vesicular

187. Patient admitted hospital with Disuse syndrome. What should the nurse instruct him
regarding disease?
A. Instruct patient to use drug timely
B. Encourage patient to walk every 2 hours in room ✅

188. What is symptoms of fluid folium excess?


A.Crackle lung sound ✅ B.Impaired skin
integrity

‫أ‪ /‬ﻣﺣﻣد ﺳﺎﻣﻲ ‪// 2996699650‬‬


189. What is the Nursing diagnosis of chest radiography‫ اﺷﻌﮫ‬tuberculosis ?
A.Ineffective breathing pattern related to tuberculosis B.Infection related disease
C.Difficult breathing related to secretion
D.Ineffective breathing pattern related to secretion ✅

190. Patient has an anxiety and turned into physical symptoms‫ اﻟﻣرض ﺗﺣول اﻟﻰ‬,
What defense mechanism is this?
A. Social isolation B.Projetion C.Regresion D,Conversion ✅

191. What is the pernicious anemia‫ﻧوع ﻣن اﻻﻧﯾﻣﯾﺎ ﺳﺑﺑﮫ‬


‫ ﻋﺷﺎن ﻛذا اﻟدم ﻧﺎزل‬12 ‫ ﻧﻘص ﻓﯾﺗﺎﻣﯾن ﺑﺎء‬complications?
A.Prolonged bleeding ✅
B.Hepatomegaly

192. Patient admitted to hospital with hip fracture. The patient was submitted to surgery
with external fixation. The patient complained from pain 3 from 10 level and numbness in
limbs. What is the most appropriate nursing intervention for patient?
A.Give patient placebo drug analgesic
B. Teach patient to take 5 deep breathing exercise ✅
C. C.Give patient local anesethia D.Allow his family stay beside him

193-25-year-old man presents with a compound fracture in the left leg and profuse
bleeding What immediate action should be taken to control the bleeding?
A. Elevate the patient's leg
B. Apply pressure on the femoral artery
C. Use a tourniquet above the fracture site
D. Apply direct pressure on the fracture site ✅

194-49-year-old women presented to the Emergency Department complaint of severe


chest pain. The ECG showed that the patient myocardial infarction. The doctor ordered
the nurse to give the 800 mg of aspirin. What is the primary indication of aspirin in this
case?
A. Breaks down the thrombus
B. Decreases the formation of platelet plugs ✅
C. C.Inhibits the conversion of prothrombine to thrombin D.Interferes with vitamin k to
maintain clotting factors

195-A 57 year-old woman presents with complaints of pain in the bones of her both
hands . What type of musculoskeletal disorder is most likely?
A.Osteoarthritis. B.Rheumatic arthritis ✅
C.Osteomyelitis D.Gout

‫أ‪ /‬ﻣﺣﻣد ﺳﺎﻣﻲ ‪// 2996699650‬‬


i

196. Which of the following patient care plan is the most appropriate for a 37 years old
post appendectomy woman who is at risk of pneumonia ?
A. Restrict fluid intake
B. Teach how to use spirometer
C. C.Encourage ambulation as tolerated ✅ ‫ﻧطﻠﺑﮫ ﻣﻧﮫ ﯾﻣﺷﻲ‬
D. D.Avoid coughing and deep breathing

197. Female Patient came very weak and tired -Not take folic acid or vitamins , low
hemoglobin , anorexic , skips meals and is recieving plasma transfusion . What priority
diagnosis ?‫ﻣﺗﺎﻛل ﺧﺎﯾﻔﮫ ﺗﻣﺗن‬
A-Altered daily living due to weakness B-Disturbed metabolism due to disease process
C-Altred dietary intake due to inability to eat ✅

198. Doctor for followup and order called and nurse responded but she doesn‘t know the
doctor and all nurses are busy what to do?‫ردت ع اﻟدﻛﺗور وﺳﺋﻠﮭﺎ وم ﺗﻌرف ﺗﺟﺎوب واﻟﻣﻣرﺿﺎت‬
‫ﻣﺷﻐوﻻت ﺗﺣوﻟﮭﺎ ع اﻟﺗﺷﺟﺎرج‬
A. Take call and administer medication ordered
B. Tell the nurse caring for patient to immediately take the call

C. C.Transfer call to patient room and have the charge nurse take the call ✅

199. caridac tamponade typical signs and symptoms ?


‫ اﻟﻐﺷﺎء اﻟﻣﺣﯾط ﺑﺎﻟﻘﻠب ﻣﺎﻟﯾﺎن ﻣوﯾﮫ‬A.Dyspnea ,
tachycardia ,chest pain
B.Cardiac muffeled murmur and hypothermia ✅ ‫ﯾﺳﻣﻊ اﻟﺻوت ﻣﻛﺗوم‬

200. Patient admitted to hospital for umbilical hernia surgery‫ ﻓﺗق‬. After doing physical
examination and Lab investigation the doctor decided to reschedule the surgery after 2
months because of result abnormalities. The informed consent already signed by the
patient. What is the best action regarding consent? ‫ﺑﯾﺳوي ﻋﻣﻠﯾﺔ ﻓﺗﺎق ﺑس اﻟدﻛﺗور اﺟﻠﮭﺎ اﻟﻧﺗﺎج ﻏﯾر‬
‫طﺑﯾﻌﯾﮫ ھل اﺧذ ﻟﮫ ﻣواﻓﻘﮫ دﯾده اﻟﻣره اﻟﺛﺎﻧﯾﮫ اﯾوه ﻻزم اﺧذ ﻣواﻓﻘﮫ‬
A. Keep the old consent it is enough
B. We need to take verbal consent later

C. C.Provide new informed consent ✅


D. D.Need to take consent from the husband

201. Patient admitted to hospital with COPD. The patient will dischrage. What is the
dischrage instructions should the nurse give?‫اش اﻓﺿل ﺗوﺟﮫ اﻋطﯾﮫ اﻟﻣرﯾض‬
A.Avoid and remove every thing that irritate him at home ✅ ‫ﯾﺗﺑﺗﻌد ﻋن ﻛل ﻣﮭﯾﺟﺎت ھذا‬
‫اﻟﻣرض‬
B.Report any signs of cyanosis
E. Provide dietary management
D.Avoid exercise

202. How to collect sputum culture?

203. Collect the sputum early morning around 30 ml

204. Instruct patient to deep breathing then cough out slowly in cup ✅

// 2996699650 ‫ ﻣﺣﻣد ﺳﺎﻣﻲ‬/‫أ‬

203-A 71 year-old woman who resides in a long-term nursing home fell while walking
down stairs;‫ ﻛﺎﻧت ﺗﻣﺷﻲ وطﺎﺣت وھﻲ واﻋﯾﮫ‬. The attending nurse arrives to find the patient
sitting motionless on the stairs. She is alert and oriented but wishes to rest. While she
rests, the nurse reviews the chart and notes that her medication regimen includes
metformin, loratadine, warfarin‫ ﻣﺎﺧذ ﻣﺿﺎد ﻟﺗﺧﺛر‬and diclofenac. Which of the following sign
most concern and should the nurse be alert? A.Bleeding
B.Bone fracture
C.Brain concussion
‫✅ ﺻﺎر ﻋﻧدھﺎ ارﺗﺟﺎج‬
D.Hypoglycemia

204-A 65 year-old woman presents to her care provider with complaints of bright red -15
blood in the stool, a loss of appetite, a feeling of fullness and fatigue. She has lost five
kilograms in the past three weeks without dieting. A faecal occult blood test is positive
and the patient scheduled for an additional screening test. Which of the following
?screening tests is the most appropriate
A. barium enema B. colonoscopy ✅
C. endoscopy
D. computed tomography scan

205-Patient with abdominal pain and abdominal distention and unable to pass stool.
When preforming rectal examination there is no stool and rectum clean. After that the
nurse inserted nasogastric tube for patient. What is the purpose for Nasogastric tube For
this patient?‫اول ھدف ﻟو ﻻﻓﺎج اﻟﻐذا وﻗﺎﻓﺎج ﻏﺳﯾل ﻟﻠﻣﻌده‬
A.obstruction B.enteral feeding
C.decompression ✅ ‫ﺗﺧﻔض اﻟﺿﻐط‬

Which of the following risk can be determined by Alpha fetoprotein analysis screening
test ?‫ﻟﯾﮫ ﻧﺳوي ھذا اﻟﺗﺣﻠﯾل اذا ﻋﻧده دﻓﻛت ف اﻻﻋﺻﺎب‬
A.Neural tube defects ✅
B.Placental insufficiency C.Hydrous fatalism D.Intra uterine growth
retardation

206-A lot of theories took about early skin contact in 3rd stage of labour what is the
purpose from early skin to skin contact between mother and child? A-at warm baby
B-improve bonding ✅
C-improve uterine contraction.
D-D-decrease maternal pulse rate

207-A nurse is preparing to administer 25 mg iron dextran inject patient with iron
deficiency anemia .the nurse knows this d to subcutaneous tissue and wants to
administer the drug safely which of the best administration techniques ?
A.Z-track
✅ B.deep
im
C. use large gauge

D.insert needle at 45 angle

// 2996699650 ‫ ﻣﺣﻣد ﺳﺎﻣﻲ‬/‫أ‬


208. Patient came to ER with burn on his face and chest. What is the first action for
nurse?
A.Access I. V line B.Give oxygen ✅

209. In the middle of providing care for the patient by the nurse. Father of child came to
her from another room and told her that his child has sever abdominal pain. What should
the nurse do?‫ﺟﺎ اﺑو اﻟوﻟد ﯾﻘول وﻟدي ﻋﻧده اﻟم ف ﺑطﻧﮫ ﺗﺗرك اﻟﻣرﯾض اﻟﻠﻲ ﻋﻧدھﺎ وﺗروح ﻟﮫ‬
A.Ask him to wait for 10 minutes B.Leave the patient and go to child ✅ C.Go to
room after the shift

210. Which of the following ethical principle define that the nurse should not allowing
personal beliefs or feelings or pressure from others or patients to affect decisions that
are made?‫اﻧو م ﺗﺧﻠﻲ ﻣﻌﺗﻘداﺗﺎت اﻟﻧﺎس اﻟﻠﻲ ﺣول ﺗﻐﯾر ﻣﻌﺗﻘداﺗﻲ‬
A. Subjectivity
B. Objectivity ✅
C. Reliability
D. Validity

211-A nurse is trying to secure a peripheral V access in a traumatic patient who is


deteriorating. The nurse has two failed attempts what
should the nurse do next?
A. Activate code
B. Use Interosseous needle
C. Consider arterial line
D. Call another nurse who is more experienced

212. During a CPR of an admitted patient in cardiac arrest, a family member tasks the
unit nurse to be at the bedside and say final words to their loved one. The nurse explains
to the family member that the scene is very disturbing and the medical team is doing its
job. \the family member still insists in witnessing the resuscitation efforts What would be
the most appropriate action by the unit nurse? A. Allow family member to be at bedside
B. Wait and ask permission from team leader
C. Call security to escort family member out
D. Refuse because there is not enough space in the room

// 2996699650 ‫ ﻣﺣﻣد ﺳﺎﻣﻲ‬/‫أ‬

213. The American Academy of pediatrics‫ ﻟو ﺟﺎ ﺑداﻟﮭﺎ ﺑﯾﻛﺗﯾرﯾﺎ ﺗﻛون ﺑﻲ‬suggests that
removal of the tonsils under certain conditions which of the following meets these
conditions?
A. 5 times viral tonsillitis per year
B. Infrequent snoring and nasal quality
C. Three times bacterial tonsillitis per 3 years ago
D. Tonsillitis accompanied by adenoid inflammation

214-A 30-year-old women was admitted with ectopic pregnancy on the sixth gestational
week. The patient was scheduled for resection of the involved fallopian tube with end to
end anastomosis which is the initial nursing diagnosis for this patient?
A. Grieving
B. Acute pain
C. Hyperthermia
D. Knowledge deficit

215. What is the most appropriate blood product for a patient with hemorrhage due to
upper GI bleeding?
A. Plasma
B. Whole blood
C. packed red cell
D. Serum albumin

216. A 40-year-old man is admitted to a Coronary pain. The ECG has normal sinus
rhythm with leads V1-V4. Blood pressure 123/69 mmHg what is the most likely nursing
diagnosis?
A. Acute chest pain
B. Myocardial in fraction
C. Decreased cardiac output
D. Ineffective tissue perfusion

217. A 75 year-old bedridden patient is hosp Medical ward for the treatment of her in
Area. Her wound is infected with multiple Longer than normal time to treat. She crying
spells and is talking about death which of the following nursing diagnosis must be
A. Health and wellness
B. Coping mechanism
C. Self-perception
D. Belief system

// 2996699650 ‫ ﻣﺣﻣد ﺳﺎﻣﻲ‬/‫أ‬

218. Nurse Manager prepares unit clinical operational plan what is top priority in the
plan?
A. Infection control
B. Staff orientation
C. Quality projects
D. Safe patient care

Pediatric
1. A healthy baby came to clinic with jaundice ‫ ﻟون اﻟﺟﻠد اﺻﻔر اﻟﺗﺣﻠﯾل ھو اﻟﻠﯾروﺑﯾن‬. What is the first action
for the nurse?
A. Ask about time of appearance of jaundice ✅ ‫اﺳﺎﻟﮫ ﻣﺗﻰ ﺑدا ﻣﻌﺎك‬
B. Assess the billirubin level by test
C. Assess hemoglobin for mother

2 .Child admitted to hospital. He was axnxious and crying too much. What is the most
appropriate cause?
A. Unfamiliar environment, and anxiety ✅ ‫ﺧﺎﯾف ﻣن اﻟﻣﻛﺎن‬
B. Pain and Fear from staff

.3 . Child came to ER with some bruising and discoloration in the skin. They try to took with him
but not responding or talking with anyone. What should the nurse suspect the child have?
A. Fear from people
B. Physical abuse ✅ ‫ﻣﻌﻧف ﺗﻌﻧﯾف ﺑدﻧﻲ‬

4. Child complaining Wheezing, crackle and nasal flaring‫ اﺣﺗﻘﺎن ف اﻻﻧف‬. The doctor ordered
Pulmicort 0.5 mg and 1 mg salbutamol. What is the most appropriate cause or What should
the nurse expect diagnosis ?
A. Viral bronchitis ✅
B. Asthma
C. Rhinitis
D. Becausehisparentsaresmoking

5. For the baby immediately after head delivery to expect good crying . What should the nurse
provide ?
A. Endotrachial tube
B. Suction secretion from nose and mouth✅
C. Warm bath
D. Slapbabyonhisbuttocks

// 2996699650 ‫ ﻣﺣﻣد ﺳﺎﻣﻲ‬/‫أ‬

6. Neonate in NICU with spina bifida. What is intervention regarding feeding ?‫ﯾﺳﺎل ﻋن اﻟﺑزوﺷﯾن‬
A. Give feeding at scheduled time ✅
B. Limit feeding for neonate
C. Stop feeding. When baby feel back pain during feeding

7.3 months baby diagnosed with Hirschsprung disease what is the disease?
a- Symptoms occur after 6 months
b. affectbothsmallandlargeintestine
c. absences of prestalisis movement in the distal part of large intestine /colon ✅
d. telescopingoftheintestine

8. The mother and her husband are sickle cell anemia carrier‫ ﺣﺎﻣﻠﯾن‬. What is the child percentage
to be carrier?
A. 25% B. 50%✅ C. 75% D. 100%

9. What is the development of six months infant?


A. Startle reflex improve
B. Move from prone to supine ✅ ‫ﯾﺗﻘﻠب‬
C. Get ready to crawling

10.A child with deformity (broken) nose, the child went to the school and his friends find this
funny,‫ اﻻطﻔﺎل ف اﻟﻣدرﺳﮫ ﯾﺿﺣﻛوا ﻋﻠﯾﮫ اﻟﻣﻣرﺿﮫ ﺗﻘول ﺑﻌد اﻟﻌﻣﻠﯾﮫ ﻛل ﺷﻲ ﺗﻣﺎم ﺗﻌطﯾﮫ‬the child was upset and went
to the nurse in the school and told him, he will stop coming to school, the nurse tokes a paper
and draw the child face and nose and tell him that ‗he will look like them after the procedure‘.
The step the nurse perform is called?
A. Self conception
B. Self confidence ✅
C. Self esteem

11. When administering an intramuscular injection to an infant, which of the following sites
appropriate for the nurse to use?
a. Rectus femoris ✅
b. Deltoid
c. Dorsogluteal
d. Ventrogluteal

12. The LPN is preparing to administer an injection of vitamin K to the newborn. The nurse
should administer the injection in the:
A. Dorsogluteal muscle.
B. Rectus femoris muscle.
C. Vastus lateralis muscle. ✅ ‫ﻻﻧﮭﺎ اھم ﻋﺿﻠﮫ ﺑﻌدھﺎ ﺑﻲ‬

// 2996699650 ‫ ﻣﺣﻣد ﺳﺎﻣﻲ‬/‫أ‬

D. Deltoidmuscle.

13.A 3 year is brought by the mother to the emergency Department with fever diarrhea and
vomiting. She passed four loose motions and three vomiting the last 24 hours , she is anorexic,
irritable , has dry lips and moderate skin turgor. She is given ORS to drink, but she refused it
after the first stip. O2 sat 96 HR 36 TEM 38.8 What is the immediate nursing intervention is
required to encourage the baby to drink the ORS?
A. Give In a cup once cold a day
B. Help her to drink with a syringe
C. Give in a small amount frequently ✅ ‫ﻣو راﺿﻲ ﯾﺷرب ﻧﻌطﯾﮫ ﻛﻣﯾﮫ ﻗﻠﯾﻠﮫ‬
D. Engageinplayingandhelptoherdrinkit

14.A child is treated for bacterial meningitis with an intravenous antimicrobial agent. Which of
the following BEST indicates
effectiveness of the treatment?‫اي ﻣن اﻻﺟﺎﺑﺎت ﺗدل ع ﻛﻔﺎﯾﺔ اﻟﻌﻼج‬
a. Increased appetite
b. Low Temperature ✅
c. Decrease pulse
d. Nodul of skin dissapear

15.The nurse have been teaching a new mother how to feed was born with a cleft lip and palate
before surgical repair of Which of the following action from the mother indicate teaching has
been successful ?
A. burping the baby frequently
B. Prevent the infant from crying
C. Placing the baby flat during feeding
D. Keep the infant prone following feedings

16) An autistic child makes no eye contact, unresponsive to continuously spins, twist and head
bang which of the following is the priority nursing diagnosis?
A. Risk of injury related to head banging
B. Impaired verbal communication related to physical
C. Personal identity disorder related to poor ego develop
D. Social isolation related to unresponsiveness towards

17.A 1-year-old girl admitted to pediatric medical unit significant weight loss, diminished mid-
arm circumference diarrhea, and red hair which of the following type of malnutrition do the nurse
suggest
A. Marasmus
B. Spitting up
C. Kwashiorkor
D. Rickets

// 2996699650 ‫ ﻣﺣﻣد ﺳﺎﻣﻲ‬/‫أ‬

18.A four-year-old child is seen in the Emergency Department fracture of the left arm. The x-ray
examination showed and healed bones‫ ﻛﺳور ﺳﺎﺑﻘﮫ ﻣو ﻣﺟﺑره‬. What is your immediate action?
A. Call social services to immediately arrange foster care
B. Ask the child about the previous accidents and mana
C. Report the child abuse to the local authorities
D. Try to establish rapport and trust with the child‘sfamily

19. A 5-year-old child is seen in the primary care clinic with


headache, malaise for about 2 days and today he has a fluids for
how long the child should be isolated after formation of?
A. 2 days I
B. 6 days
C. 10 days
D. 15 days

20. A nurse in the Neonatal Intensive Care Unit is caring for premature newborn, is diagnosed
with Respiratory Distre(RDS) and the doctor ordered administrating surfactant Surfactant
should be given by which of the following routes?
A. Intravenous
B. Subcutaneous
C. Intramuscular
D. Endotracheal

21. A 1-year-old child was seen to the Emergency Department abdominal pain, palpable sauge-
shaped mass, and Intussusception is suspected which of the following the best diagnostic
evaluation is to?
A. X-ray
B. endoscopy
C. Rectal biopsy
D. Ultrasonography

22. A term baby boy has diagnosed with Down syndrome. Physical examination revealed
flattened nose, low set ears, upward slanting eyes single palmer crease Which of the
following is the most common congenital anomaly associated with the this disease?
A. Developmental dysplasia of hip (DDH)
B. Congenital heart disease
C. Hypospadias
D. Pyloric stenosis

23. A 7-month-old infant seen in the Emergency Department suffering from episodes of severe
abdominal pain , and the infan‘s stool become like red jelly. Abdominal examination revealed
palpable sausage-shaped mass in the right upper quadrant which of the following is the best
diagnosis?
A. Hirschsprung disease
B. Hypertrophic pyloric stenosis
C. Infant colic
D. Intussuception

// 2996699650 ‫ ﻣﺣﻣد ﺳﺎﻣﻲ‬/‫أ‬

24. A 2-day-old newborn is admitted to the nursery. While the nurse is administrating oral
feeding, the milk returns through the child‘s nose and Mouth and the infant become
cyanotic which of the following condition the newborn should have
O A. Anorectal malformation
O B. Tracheoesophageal
fistula‫ھﻧﺎ اﻟﺣﻠﯾب م ﯾطﻠﻊ ﻣن اﻻﻧف‬
O C. Cleft lip and palate
O D. Cardiac condition

25. Five year‘s old patient was brought to emergency room with several bruises on his body but
showed fractured right forearm. He had no signs of pain while palpating them. He seemed
scared and didn't answer any questions asked. Why should the nurse discuss this manager
with the nurse?
A-care continuity
B-abuse role out
C-support psychological
D-management fracture

26.A 9-year-old child is admitted to pediatric word diagnosed with Glomerulonephritis (AGN).
The mother ask the nurse why you the blood pressure frequently which of the following the
nurse should be respond based on knowledge.
A. Acute hypertension must be anticipated and identified
B. Hypotension leading to sudden shock
C. Blood pressure fluctuations are a common side effect of therapy
D. Blood pressure fluctuation are a sign that the condition has become chronic

27.A 2-month-old infant with cleft lip is seen in the primary health care to get the regular vaccine
of 2 months. The mother asked proper time for the corrective cleft lip surgery of her infant.
Which of following is the best nurse response?
A. No specific age for repair of cleft lip
B. It is too late, repair should be done immediate after delivery
C. The age of 2 months old is the time for repair
D. The proper time for repair after the age of one year,

28. The nurse is receiving a child postoperative tonsillectomy. Which


of the following nursing assessment is suitable for the postop care?
A. Encourage the child to cough spontaneously
B. Observe for subtle signs of hemorrhage
C. Place the child in the prone position
D. Suction the mouth to clear the airway Because of violent behavior

// 2996699650 ‫ ﻣﺣﻣد ﺳﺎﻣﻲ‬/‫أ‬

29.A 10-year-old girl presents to the Emergency Room (ER) with Leaft pain. On assessment,
you noticed that when you palpate the leaft of the child‘s abdomen , the child feels pain in the
rigquadrant. Which of the following is the name of this sign?
A. Rebound tenderness
B. McBurney sign
C. Roving‘s sign
D. Obdurate sign

30. The nurse is assessing a 2 -years-old child with Wilms sutrgery which of the following
should the nurse avoid?
A. Putting the child in lateral position
B. Palpating the child's abdomen
C. Putting the child in a private room
D. Provide mouth hygiene 30 minutes after meal

31. A 5-month-old boy has been vomiting green colored vomit He has intermittent abdominal
pain during which he draws his chest, turns pale and cries forcefully. On observation, the in
the stool which has a jelly-like consistency. Abdominal pal a long. Tube-like mass. There is
no fever, rash nor diarrhea are hyperactive in all quadrants? Which is the most likely form of
initial treatment?
A. Manual manipulation
B. Surgical resection
C. Normal saline enema
D. Laparoscopy

32. A child was admitted to the hospital three hours ago with a injury. The child responds
appropriately, but sluggishly ‫ ﺧﻣول‬to drifts in and out of sleep which of the following best
describes this patient's level of?
A. Lethargic
B. Obtunded
C. Comatose
D. Semicomatose

33.A mother brought her 6-month-old healthy infant to the well- baby clinic which immunization
should the nurse anticipate to administer as per World Health Organization's recommendation?
A. Varicella (Chicken pox)
B. Rotavirus and hepatitis
C. Measles, Mumps, Rubella
D. Diphtheria, Tetanus and pertussis ,BCG

34. a nurse is caring for a child with traction of fractured bone. In the chart, a doctor has placed
a reminder to maintain even and constant traction what would be the most likely
understanding of the nurse for this order?
A. Add or remove weights every other day

// 2996699650 ‫ ﻣﺣﻣد ﺳﺎﻣﻲ‬/‫أ‬

B. Allow weights to hang free continuously‫ﻧﺣطﮫ اﻻوزان ﺑﺷك ﻣﺳﺗﻣر‬


C. Elevate head and foot of the bed alternatively when in pain
D. Allow weights to hang free every 12 hours to achieve good circulation

35. The nurse is assessing a child who has Tetralogy of Fallot observed that the child is having
clubbing in his fingernails ‫ ﺿرب ف اظﻔره‬which of the flowing might be the reason for this
clubbing?
A. Prolonged tissue hypoxia
B. Delayed physical growth
C. Inactive bone marrow
D. Pulmonary fibrosis

36) In determining the one minute Apgar score of a male in assesses a heart rate of 120 beats
per minute and respiratory rate 44 per minute. He has flaccid muscle tone with slight flex44
resistance to straightening. He has a loud cry with color is acrocyanotic, what is the APGAR
score for the infant?
A. 7
B. 8
C.
D. 9
E. 10

37. A nurse is caring for a child with traction of fractured bone doctor has placed a reminder to
maintain even and constant what would be the most likely understanding of the nnursefo?
A. Add or remove weights every other day
B. Allow weights to hang free continuously
C. Elevate head and foot of the bed alternatively when i
D. Allow weights to hang free every 12 hours to achieve circulation

38. A 9-year-old child is postoperative after tonsillectomy the nurse should ask the parents to
give the child which of food after discharge from the hospital?
A. Meat and rice
B. Hot dog and potato chips
C. Mashed potatoes and soup
D. Cucumbers and tomato salad

39. A 7-year-old child is admitted to the Emergency Department, injury the child is oriented to 4
the place and person and time spontaneously 5 obeys commands.6 The nurse is doing aped
Coma Scale PGCS). Which of the following score the nurse should record?
A. 3
B. 8
C. 12
D. 15

// 2996699650 ‫ ﻣﺣﻣد ﺳﺎﻣﻲ‬/‫أ‬

40) A 5-year-old child was admitted with Nephrotic Syndrome. A nurse noticed that the child has
slight facial puffiness with mild pitting edema on his hands and feet. There was no distended
abdomen Which type of diet the nurse should order for the child?
A. High protein, high salt diet
B. Low protein, low fibre diet
C. Low protein, normal salt diet
D. Normal protein, low salt diet

41. A nurse documented assessment on a newborn as listed (see table)


Indicator Heart rate Respiratory rate Muscle tone Reflex irritability Colour Points
less than 98 /min 28 /min irregular Minimal flexion of the extremities
Grimace Body pink, extremities blue 1 What is the total Apgar Sore?
A. 0
B. 3
C. 5
D. 7

42) A toddler is seen in the Emergency Room with a history of high 3 temperature for 5 days.
He had watery eyes and a cough for the last days, and then he developed a rash all over
the body which started in his face and spread down to his body. The mother did not
remember the vaccination schedule of her son. For how long the child should be isolated?
A. 5 days
B. 10 days
C. 15 days
D. 20 days

43) Infant is born with spinal bifida. Which of the following complication is always found in these
Infants?
A. Hydrocephalus ‫ﺣﺟم اﻟدﻣﺎغ اﻋﻠﻰ ﻣن اﻟطﺑﯾﻌﻲ‬
B. Craniosynostosis
C. Meningitis
D. Cerebral policy

44. A nurse is performing physical examination on a newborn. She that the baby

has developed cephalohematoma. This baby is at risk of which of the

following?

A. Sudden death

B. Pathological jaundice ✅ ‫اﻟﺟﻧدس ﻟﮭﺎ ﻟﻧوﻋﻲ ﺑﺎﺛو ﻓﯾﺳوﻟوﺟﻲ ھﯾﻣﺎﺗوﻣﺎ ع اﻟدﻣﺎع ھﺎ ع اﻟرﯾﺑﯾون‬

C. Infected umbilical cot

D. Increased intracranial pressure

// 2996699650 ‫ ﻣﺣﻣد ﺳﺎﻣﻲ‬/‫أ‬

46. side effects of bCG vaccination


A-cold and small scar✅
B-diarrhea
C- rash for three days

47. Child with Pyloric stenosis‫ ﺗﺿﯾق اﻟﺑواﺑﮫ اﻟﻠﻲ ﺑﯾن اﻻﻣﻌﺎء واﻟدﯾودﯾﻧم‬. What is the
expected signs and symptoms postoperative?
A. Abdominal pain B. Watery stool C. Vomiting ✅ D. Urinary

48. A nurse is collecting a urine of a 4-year-old child with nephrotic syndrome.


Which of following observation about the color of the child's urine the nurse
expected to? will chart. A. Bright red B. Amber C. Dark, frothy D. Orang

49. The nurse is assessing a child (an infant) with pyloric stenosis. which of the
following is likely to note?
A. Diarrhea
B. Projectile vomiting✅
C. Swallowing difficulties
D. Currant jelly like stool

50. Child with VSD‫ ﻋﯾوب ﺧﻠﻘﯾﮫ ف اﻟﻔﻠب وش اﻻﺣظ‬. What is


blood flow characteristic? A- mixes atrium blood
B-decreased pulmonary blood flow
C-increase pulmonary blood flow

51. Infant 11 month On ECG heart rate 240 b/m. Embedded ‫ ﻣﺗﺿﯾﻘﮫ‬in the QRS
complexes and absent p wave. What is the expected diagnosis for the infant???
A. Ventricular tachycardia (VT) B. Bradycardia C. Supraventricular tachycardia
(SVT) D. Atrial fibrillation Answer: C

52. Patient has cystic fibrosis and the nurse wants to perform postural
drainage‫ ﻧﺿرب ع ﺻدره ﻧطﻠﻊ اﻟﺑﻠﻐم‬when is the best time for posture drainage? A-Before
meals B-After meals after half an hour of meals‫ ✅ ﺑﻌد اﻻﻛل ﺑﺳﺎﻋﮫ‬C-Before sleeping
D. Afternoon Answer: B

53. A 4-month-old infant returned immediately from OR room post cleft lip repair
which intervention should be considered.?
A. Apply elbow restrain ✅
B. Apply suction when needed
C. Measure temperature
D. Put infant in prone position Answer: A

// 2996699650 ‫ ﻣﺣﻣد ﺳﺎﻣﻲ‬/‫أ‬

54. In determining the one-minute APGAR score of a male infant the nurse
assesses a heart rate of 120 beats per minute and respiratory rate of 44 per
minute. He has flaccid muscle tone with slight flexion and resistance to straightening. He
has a loud cry with colour is acrocyanotic What is the APGAR score for the infant?
A. 7
B.
8
C.9
D.10

55. A 20 weeks pregnant, primary gravid woman visits the antenatal has sickle cell
anemia trait and worried this disease transmitted to her baby which of the following
should be initial intervention?
A. Plan for the fetal genetic screening
B. Educate mother that her disease is inactive.
C. Discuss the chances of genetic disease in the fetus.
D. Gather data about the other family members having the disease.

56. month-old boy with hydrocephalus is admitted to the pediatric surgical Ward
for ventriculoperitoneal Shunt ‫( ﺗﺑوب دﺧل اﻟدﻣﺎغ ﯾﺳﺣب اﻟﺳواﺋل‬VPS) insertion. Which
of the following findings should be of the most concern when assessing the
child postoperative?
A. Sunken fontanelle and irritability
B. decreased head circumference
C. poor feeding and pupillary change
D. headache and excessive sleepiness

57. month-old boy with hydrocephalus is admitted to the pediatric surgical Ward
for ventriculoperitoneal Shunt (VPS) insertion. Which of the following findings
should be of the most concern when assessing the child postoperative?
A. Plugging fontanelle ‫ﻻﻧو ھﻧﺎ اﻟﺑﻠﺟﻧق اھم ﻣن اﻟﻔﯾدﯾﻧق ﻟو ﻟﻘﯾﻧﺎ اﻟﺑﻠﺟﻧق ﻣﻊ اﻟﻔﯾدﻧق ف اﻻﺧﺗﯾﺎرات ﻧﺣط ﺑﻠﺟﻧق‬
B. decreased head circumference
C. poor feeding and pupillary change.
D. headache and excessive sleepiness
Answer: A

58. What is the Contraindication of baby vaccine?


A. Antiemetic
B. Antibiotics
C. Steroid

// 2996699650 ‫ ﻣﺣﻣد ﺳﺎﻣﻲ‬/‫أ‬

59. Child with Pyloric stenosis‫ ﺗﺿﯾق ﺑﯾن اﻟدﯾدوﻧم واﻟﻣﻌده‬. What is the expected
signs and symptoms postoperative?
A- abdominal pain
b- watery stool
c- vomiting
d- urinary

60. mother has infant 6 months came to the clinic and afraid that her baby May will
Have meningitis as his brother already have. What should the nurse tell the mother?
1-hib vaccine can decrease the meningitis ✅ ✅
2. there are now vaccinations for all meningitis type
3. tell the mother the disease is will not come

61. 1 month-old infant is admitted to the surgical unit with hypertrophic pyloric
stenosis and scheduled for the surgery. Which of the following is the findings of
abdominal examination?
A. palpable olive-like mass in the left side
B. palpable olive-like mass in the right side ✅ ‫اول ﻋﻼﻣﮫ ﻓوﻣﯾﺗﻧق ذي اﻟﺛﺎﻧﯾﮫ‬
C. Palpable olive-like mass moved from left to right.
D. Palpable olive-like mass moved from right to left

62. Full term Newborn was delivered 2 hour ago. The nurse checks him observed.
mild peripheral cyanosis. What is the first action?
A. Call for help
B. Notify the doctor
C. Check temperature

63. what complications after cleft palate repair?


A) Teeth pain
B) Deviated septum.
C) speech
difficulty‫ﺻﻌوﺑﮫ ف اﻟﻛﻼم‬
D)recurrent tonsillitis

64. Child with Pyloric stenosis. What is the expected signs and symptoms
postoperative?
A. Abdominal pain
B. Watery stool
C. Vomiting‫ﺑﺷﻛل ﺿﻌﯾف‬
D. Urinary

// 2996699650 ‫ ﻣﺣﻣد ﺳﺎﻣﻲ‬/‫أ‬

65. Infant came to ER and during the examination, the doctor said he had spinal
bifida occult. What does the doctor know about this condition?
A. Infant has hair patch at back
B. Infant has open wound on back

66. A5 - year - old child was brought to the Emergency Room with a fractured of
forearm. He had several bruises on his body but showed no signs of pain while palpating
them. He seemed scared and did not answer any questions asked. How should the
nurse initiate therapeutic communication with the child?
A. Start interviewing
B. Encourage him to speak.
C. Explain about the fracture
D. Greet and show gentleness - nurse giveshealth.

67. 4 years old Child with cast. What is the nursing care for cast?
A. Put in hot water
B. Put Infront of fan or cooler to dry fast
C.Put in cold water to dry fast
D.Put powder on the edges of the ca to prevent itching

68. Infant has breastfeeding associated jaundice , The nurse should instruct the
mother to:
A- Stop breastfeeding for one week
B- Wean the child
C- Phototherapy
D - Increase the frequency of breastfeeding

69. 2years old child with pneumonia. what best


intervention?
A, encourage cough and deep breathing
B, Encourage exercise
C-provide high carbohydrate diet

70. 10 years boy with polyuria and dysuria after assessment diagnosed with
urinary tract infections what should do to take urine sample?
A. Increase fluid intake
B. Decrease urineintake
C.Regular intakeof fluid
D. Zero intake offluid

70. Child with recurren oitis media.What is risk factor should the nurse instruct Parent?
A. Fever
B. Respiratory tract infection .
C .Pain in ear

// 2996699650 ‫ ﻣﺣﻣد ﺳﺎﻣﻲ‬/‫أ‬

D. Foreign body

71. While a nurse is assessing an infant born 11 hours ago full term by caesarean
section, she auscultated moist lung sounds. Which of the following is the most likely
interpretation?
A. Abnormal finding
B. Normal finding
C. Pneumothorax
D. Surfactant aspiration

72. A30week gestational preterm admitted to NICU 2hours ago the neonate starts
to have grunting, nasal flaring‫ ﺷﺧﯾر‬. which of the following the nurse recognize
regarding signs and symptoms ?
A. Neonate has RDS‫ﻣﺷﺎﻛل ف اﻟﺟﮭﺎز اﻟﺗﻧﻔﺳﻲ‬
B. It is normally in the first 24 hours of birth.
C. This is not significant unless become cyanosis.
D. Neonate has hypoglycemia.

73. what is chest assessment for patient with vitamin D deficiency?


A. Dull sound
B. .Rosary‫ﺑطن ﻣﻧوخ واﻟﺻدر داﺧل‬
C. . Crackles
D. .Chest deformity

74. nurse is assessing a 5-months-old infant who was admitted to the pediatric
ward Coarctation‫ ﺗﺿﯾق ف اﻻورطﻰ‬of the Aorta (COA) Which of the following is the
most common assessment finding?
A. Cyanosis and clubbing of fingertips
B. Bounding pulse and hypotonicity
C.Cyanosis occur frequently during and after feeding
D. Blood pressure is different on the arm the leg

75. The nurse is auscultating breath sound for infant.Which of the following related
to high pitch with harsh breath sound??
A. Crackles
B. Tracheal
C. Wheezing
D. Rhonch

76. A newborn baby is being evaluated on the APGAR score immediately after his
birth. The score measures the quality of newborn's pulse rate, reflexes, muscles to
neand respiration. His APGAR score is graded as 7 out 10.What is the significance
of measuring the APGAR score immediately after the birth ?

// 2996699650 ‫ ﻣﺣﻣد ﺳﺎﻣﻲ‬/‫أ‬

A— It helps to plan the treatment for congenital diseases B


— It serves as apermanent record for the new born babies
C— It provides the bases for the comparison, a few minutes later
D— It helps identifying the abnormalities related to muscular tone

77. ThenurseAssessNeonate after delivery using Apgar score. Spontaneous


respiration Prompt respond Limited cry 98 pulse Pinkish body color excepthands
What is the expected score ?
A— 8
B— 7
C— 6
D— 5

78. A9 -year-old child is admitted to the Emergency Department injury. The child is
oriented to the place, person and time, spontaneously, obeys commands. The
nurse is doing a ped Coma Scale (PGCS).Which of the following score the nurse
should record?
A— 3 B— 8 C— 12 D— 15

79. The neonate is delivered by cesarean section the baby should be transported to
NICU. Which type of baby incubator should be used?
A - Closed box incubators
B - Portable incubators
C - Double - walled incubators
D- Servo - control incubators

80. 3 months baby diagnosed with Hirschsprung disease what is the disease?
a- Symptoms occur after 6 months
b- affect both small and large intestine
C- absences of prestalisis movement in the distal part of large intestine / colon
d- telescoping of the intestine

81. A 1-year-old girl admitted to podiatric medical unit significant weight loss,
diminished midarm. circumference diarrhea, and red hair. Which of the following type of
malnutrition do the nurse suspect?
A. Marasmus
B. Spitting up
C. Kwashiorkor
D. Rickets

82. child with sickle cell crisis and take oxygen, how to know oxygen was evective?
A-by heart rate B-pain level
C-respiratory rate ✅

// 2996699650 ‫ ﻣﺣﻣد ﺳﺎﻣﻲ‬/‫أ‬

83. baby with trachea esophageal fistula and has scheduled for surgery, how to feed
the baby before surgery?
A-gastrostomy B-Brest feeding C. NG tube ✅ D-bottle feeding

84. Which of the following hirschsprung disease causes?


A. Congenital ✅ ‫م ﯾطل اﻟﻣﻛﯾﻧﯾوم اﻟﻠﻲ ھو اﻻﺳﺗول اﻻول اول ﯾوم‬
B. Constipation
C. The child is infected at the age of 6 months

85. Child admitted to surgical ward for thyroidectomy .. Which position for child with
thyroidectomy ?
A. Semi fowler with slight flexion neck ✅
B. Supine with hyperextened neck

86. How many times shower the baby?


A.twice a week with warm water
B.Every day with warm water
C.Every day with warm water and acidic soap D.twice a week with warm water and
alkaline soap ✅

87. neonate after one day of delivery diagnosed with tracheoesophageal fistula.
What should the nurse observe for neonate? A-continuous crying ✅
B-Vomiting
A 6 months old infant mother decided to wean her child. Which of the following is the
best principle of weaning process?
A.Start the weaning process by 8 month of life B.Gradually replace one breast session at
a time ✅ C.Discontinues the nighttime feeding first
D.Allow the child to take a bottle of milk or juice bed

87-2 years old girl came to ER with perant. She experienced frequent urination. She lost
3 kg of her weight. Which of the following is the best diagnostic tests?
A.CT
B.CBC
C.Fat
D.Blood glucose level ✅

88. Child came to the ER with his parents to report as a witness about one victim ‫اﻟطﻔل‬
‫ ﺧﺎﯾف‬from their neighbours‫ اﻟﺟﯾران‬. What should the nurse consider when question the
child?
A.Developmental activity
B.His awareness of the incident ✅ ‫اﻛﺗب اﻛﺳدﯾﻧت‬

‫أ‪ /‬ﻣﺣﻣد ﺳﺎﻣﻲ ‪// 2996699650‬‬


89. Child came to ER confirmed german measles. What is the best intervention for child
?
A. Antipyretics for low grade fever ✅ B.Start I. V antibiotics C.Antihistamine for itching

90. RSV vaccine route for premature baby is ?


A-IM
B-iV
C-SC
D- Oral

91. How many week fever sustain in rheumatic fever?

A.1.week B.4 days C.5 days D.3 days

Community
1. The nurse is instructing and learning patient and his relatives how to change patient dressing.
What is the step confirm for evaluation?‫اش اﻟﺧطوه‬
A. Verblize the tools that used in dressing change
B. Verbalize the steps of dressing change oral
C. Demonstrate wound dressing and observe for them ✅ ‫ﺣﻔظ‬

2 .The community health centers want to improve condition for people with disabilities . What is
Primary prevention for disabilities patients?
A. Contact with families and supporting organization✅ ‫ﺧطﺎ‬
‫ﻣوﺟوده ف اﻟدﻓﺗر ﺣﻔظ اﻻﺟﺎﺑﮫ ﺗﻛون ادﯾﻛوﯾﺷن‬

3. The nurse is giving health education for mother about oral contraceptive pills. The mother
asked the nurse about side effects. What should the nurse response?
A. Don't ask
B. Do you want to clarify about something
C. Nausea, water retention and weight gain ✅

4. The nurse is providing Session about ( disease process, coping, dietary management) for
patient. Which is the initial important long term goal?
A. Increase understanding of disease process
B. Have a healthy life style ✅
C. Increase coping

// 2996699650 ‫ ﻣﺣﻣد ﺳﺎﻣﻲ‬/‫أ‬

D. Dietarymanagement

5. There is single parents. One of both has one child. They will married. What is the type of
family?‫ﻣطﻠق وﻣطﻠﻘﮭﻛل واﺣد ﺑﯾﺗزوج وﻋﻧﮭم ﻋﯾﺎل ﻧوع اﻟﻌﺎﯾﻠﮫ‬
A. Nuclear B. Extended C. Blended ✅

6. Community health nurse is making preparation for crisis plan. What is the most accurate
information to make a disaster plan in community ?‫ﺟﺎﻟﺳﮫ ﺗﺷرح ﺧطﺔ ﻟﻼزﻣﺎت أي ﻣن اﻻﺟﺎﺑﺎت ﺗﻛون ﻣن‬
‫ﺿﻣن ھﺎﻟﺧطﮫ‬
A. Check stored water every year and change it if expired
B. Store bag with 3 gallons of water for each person for 3 days ( 1 gallon per day for each
person). ✅
C. Inform that animals don't need plan. It can survive
D. PreparebagandStoringoffoodwaterandotherstufffor7days

7. Which issue would a unit quality improvement committee address?‫ﻛﯾف اﺣﺳن ﺟودة اﻟﻘﺳم اﻟﻠﻲ اﻧﺎ ﻓﯾﮫ‬
A. A 10% decrease in client satisfaction in the registration process
B. A nurse who made 3 medication errors in the past quarter
C. An increase in catheter-associated urinary tract infections ✅
D. Staffperceptionofhospitallaboratorypersonnelincivility

8. Interdisciplinary client care rounds and hand-off communication are examples of strategies
used to improve communication in health care settings. What is the most important outcome
of effective communication among care givers?‫ﻛﯾف ﯾﻘﻘﻠﻠو اﻻﺧطﺎء ف اﻻﺗﺻﺎل‬
A. Decreased length of hospital stay
B. Less obvious needs of clients met accordingly C. Properly educated clients
D. Reduced number of medical errors✅ ‫اﻻﺧطﺎء ﺗﻘل‬

9. A physically handicapped young woman uses a wheel chair to her problem. She had a
neighbor who repeatedly tell her that did a fault and for that she has been punished from
God. Which model of disability does this attitude belongs to?
A. Moral
B. Medical
C. Disability
D. Rehabilitation -

10. In a discharge plan of newly diagnosed patient with Hepatitis C, encourages the patient and
family to cope with the disease pro includes taking medication regularly, good nutrition, and
appointments. Which of the following best explains the nurse‘s role the family?
A. Help understand patient‘s condition
B. Ask to respect patient‘s condition
C. Explain the disease process

// 2996699650 ‫ ﻣﺣﻣد ﺳﺎﻣﻲ‬/‫أ‬

D. Guide to be supportive

11. The nurse is providing health education for parent has child after shunt. What is the teaching
should included to family ?
A. Increase protein diet
B. Restrict activity ✅ ‫ﻧوﻗف اﻻﻧﺷطﮫ‬

12. Nurse community visit school students what the most important topic to focus on.
A. Sport safety
B. Bicycle safety✅
C. Road safety

13. a new community nurse in the rural clinics has to have good background about the health
status of the community in order to assess their needs What is the appropriate way to
assess the health status of the community?‫ﻛﯾف اﻗﯾم اﻟﻣﺟﺗﻣﻊ‬
A. Home visit
B. Community assembly
C. Mass information campaign
D. Community health assessments

14. Mother of nine children, three of them with congenital an down syndrome; she is a primary
school graduate, with status. She is not using any method of family planning. So Health
care nurse has referred her for counselling which of the following must be focused on by the
community nurse to provide an effective health education?
A. Educate regardless realistic objectives
B. Use clear and concise language
C. Use scientific terms during explanation
D. Explain the negative consequences in the family

15. There is a new community nurse assigned to work in a Makkah, one of the most
multicultural cities in Saudi Arab has to interact with multicultural population every day what
is the most important step the nurse should do before with the clients?
A. Asking the client about his/her cultural background
B. Conducting an appropriate culturological assessment
C. Ensuring that the client has his/her a privacy
D. Looking at the client's file to take history

16 . Home Health Care Department is newly established ar a hospital. At launch, nurses asked
the charge nurse about work at this department which of the following is the first step of home
health care?
A. Referral
B. Planning
C. Home visit
D. Assessment ‫ﻟﻣن ﯾﻛون ف اﻟﻣﺳﺗﺷﻔﻰ‬

// 2996699650 ‫ ﻣﺣﻣد ﺳﺎﻣﻲ‬/‫أ‬

17. A Community nursing nurse is working with a family in their home the parents complain that
their eight year-old son is "wild" and that he never listens to them. They become upset at his
antics to gain attention from the nurse and send him out of the room. Which of the following
responses from the nurse is the most appropriate?‫ﻋﻣره ﺛﻣﻧﯾﮫ ﺳﻧوات وﻣﺷﺎﻏب اﻻﺟﺎﺑﮫ اﻋﻠم اﻻھل ﻛﯾف‬
‫ﯾﺗﻌﺎﻣﻠو ﻣﻌﺎه‬
A. Tha was the right thing, it teaches him who is in charge
B. Don‘t worry, he is a boy and with time, he will grow out of this
C. Let‘s talk about how we can teach your child the right behavior and your expectations of
him✅
D. You are only reinforcing his behavior when you do That. Try putting him in time out instead

18. A Community nursing nurse is working with a family in their home the parents complain that
their eight year-old son is "wild" and that he never listens to them. They become upset at his
antics to gain attention from the nurse and send him out of the room. Which of the following
responses from the nurse is the most appropriate?
A. Tha was the right thing, it teaches him who is in charge
B. Don‘t worry, he is a boy and with time, he will grow out of this
C. Let‘s talk about how we can teach your child the right behavior and your expectations of
him✅
D. You are only reinforcing his behavior when you do
That. Try putting him in time out instead

19. The community nurse was instructing for two partners will marry and advise

them about health life and obligations. What is this considering?

A. Counseling
B. Education
C. Screening

20. Which step in community organizing involves training of potential leaders in the community?‫اش‬
‫اﻟﺧطوه اﻟﻠﻲ ﻧﺳوﯾﮭﺎ ﻋﺷﺎن ﻧﻧظم اﻟﻣﺟﺗﻣﻊ اﻻﺟﺎﺑﮫ دي ﻋﺷﺎن اﻟﺗراﯾﻧﯾﻧق‬

A. Integration
B. Community organization C. Community study
D. Core group formation✅

21. In which step are plans formulated for solving community problems?‫اي ﺧطوه ﻧﺷوف ﻓﯾﮭﺎ ﺣﻠول اﻟﻣﺷﺎﻛل‬
‫ف اﻟﻣﺟﺗﻣﻊ‬

B. Mobilization
C. Community organization ✅
D. Follow-up/extension
E. Core group formation

‫أ‪ /‬ﻣﺣﻣد ﺳﺎﻣﻲ ‪// 2996699650‬‬


22. The community health nurse visit the patient to provide health education about colostomy
care,. She found the patient with his relatives and will go outside. What should the nurse
do?‫راﺣت ﺗﺳوي ﻟو ادﯾﻛﯾﺷن ﻗﺑل ﯾطﻠﻊ‬

A. Write the patieng refuse the visit


B. Don't go out, you will be late
C. Arrange for another visit ✅

23. Community health nurse visited alzheimer's disease patient. The patient complained from
bruises and many injuries. The caregiver denied any fallness caused for patient. What is
the nurse response?

A. Restrain the patient

B. Blame to patient that he is the reason

C. The cause is a cognitive impairement

D. Arrange room and provide light night in room ✅ ‫ﻋﺷﺎن اذا ﺻﺣﻲ م ﯾطﯾﺢ‬

24. Community health nurse is providing Education for secondery schoole girls. Which of the
following is the best topic to discuss?‫ﺗﻌﻠم طﻼب ﻣﺗوﺳط اش اﻟﻣوﺿوع اﻟﻣﻔروض ﺗﻧﺎﻗﺷﮭم ﻓﯾﮫ‬

A.Annualy examination importance

B.Personal Hygiene

C.Ovarian cancer

25. Community health nurse visited patient in home with alzheimer's disease. The patient
complained from bruises and several injuries on his body. The community nurse noticed
patient fall and The caregiver denied any fallness occurred for the patient. What should the
nurse do?

A.Provide restrain to the patient


B.Blame the patient that he is the reason
C.Arrange the room and provide nightlight
D.The cause of fall is cognitive impairment

26. Patient told nurse researcher he wants to stop the research her response?‫م ﯾﺑﻐﻰ ﯾﻛﻣل ف اﻟﺑﺣث‬

A.you can stop at any time ✅


B.You can‘t stop
C.You must have good reason to stop

// 2996699650 ‫ ﻣﺣﻣد ﺳﺎﻣﻲ‬/‫أ‬

Management
1 .During a night shift a medical doctor complains of back pain and asks the t nurse to give him
morphine 5 mg IM. Which of the following actions indicates professionalism in handing the ation
by the nurse?
A. Call another doctor to manage
B. Refer him to Emergency Room ✅
C. Administer morphine to doctor
D. Askhimtowriteaprescriptionfirst

2. nurse want to increase size of staff next year , what is the first thing by human resources?
Downsizing Recruitment ✅ ‫اﻗل ﻧﺳﺑﺔ اﻟﻣﺗﻘﺎﻋدﯾن‬

3. The hospital managers observed that staff using personal smart phones too much during work
time to get information related job. What should hospital do?‫ﻣﺳؤول اﻟﻣﺳﺗﺷﻔﻰ ﻻﺣط اﻧﮭم اﺳﺗﺧدﻣوا اﻟﺟوال‬
‫اﺛﻧﺎء اﻟﻌﻣل‬
A. Prevent staff to accompany the phones or bring inside work area
B. Provide staff with another phones from the hospital
C. Set new policy to reduce staff using smart phones ✅

4. The doctor was busy. He requested from the nurse to see certain patient. What is this called?
‫اﻟطﺑﯾب ﻣﺷﻐول وطﻠب ﻣﻧﻲ اﺷوف ﻣرﯾض ﻣﻌﯾن‬
A. Practitioner
B. Responsibility ✅

5. While the nurse checking patient file to transfer him to operation room. She found that the
doctor didn't get the patient's signature in informed consent. What is this consider?‫اﻟطﺑﯾب م‬
‫وﻗﻊ اﻟﻛوﻧﺳﺑت‬
A. Negligence ✅ ‫اھﻣﺎل‬
B. Malpractice

6. Which type of study design provides the strongest evidence?‫اي ﻣن اﻟﺑﺣوث اﻗوى اﻟﻌﺷواﺋﻲ‬
A. Qualitative study
B. Randomized control trial ✅
C. Systematic review of descriptive studies
D. Systematicreviewofcorrelationalstudies

7. A new nurse threw a needle into the waste basket. When asked action, the new nurse admits
the mistake. Which professional act best describes the nurse‘s response?
A. Responsibility
B. Accountability
C. Assertiveness
D. Autonomy

// 2996699650 ‫ ﻣﺣﻣد ﺳﺎﻣﻲ‬/‫أ‬

8. The intensive care unit nurse manager plans to delegates a nurse. What is indicated for a
successful delegation?‫اي ﻣن اﻻﺟﺑﺎت ﺗدل ع اﻟﺗﻔوض اﻟﻧﺎﺟﺢ‬
A. Nurse Manager supervises nurse.
B. Nurse has authority to change task.
C. Nurse Manager checks task sometimes.
D. Nurse Manager asks another nurse to check task

9. A patient fell in bathroom and his left leg was fractured, in order to communicate information
about the patient to next shift.Which of the following documentation should be used by the
nurse at the end of the shift?‫ﺗﺑدﯾل اﻟﺷﻔت اش ﻧﻌﺑﻲ اﻟورﻗﮫ اﺳﻣﮭﺎ‬
A.Kardex record✅
. B.Assignment record
C.Shift report
. D.Incident report

10. A nurse researcher wrote in her explanatory statement: "The researcher will - 35 Use a
questionnaire to measure nursing job satisfaction," which ethics principle is the
researcher addressing here? ‫ﯾﻛﻣﻠوا ﺷﮭﺎدات ﻋﻠﯾﺎ ﯾﺟﯾﺑوا اﻟﺑﺣث ﻻزم اﺟﺎوب ﺻﺢ ﻋﺷﺎن اﺣﻘق اﻟﻌداﻟﮫ‬
A. Justice ✅ ‫اﻟﻌداﻟﮫ‬
B. Beneficence
C. Confidentiality
D. Non-maleficencge

11) The nurse manager received complaints from some nursing to improper work distribution
and discrimination how should the nurse manager handle the situation?‫اﻧﺎ اﺳﺗﻠم ﺛﻼث ﺣﺎﻻت وﻧﺎس‬
‫ﺣﺎﻟﺗﯾن وش اﺳوي اﺗﺣﻘق‬
A. Investigate the complaints
B. Individual counselling
C. Identify list of problems
D. Continue to observe

12. After accessing patients' medical records, which behavior nurse shows that patient‘s
confidentiality has been breached?‫اش اﻻﺟﺎﺑﮫ ﺗدل ع اﻧﺗﮭﺎك ﺧﺻوﺻﯾﺔ اﻟﻣرﯾض‬
A. Reviews patients medical record‫؟‬
B. Read patients care plan
C. Disclosing patient‘s information‫ﻣﺷﺎرﻛﺔ ﻣﻌﻠوﻣﺎت‬
D. Documents medication administered

13. In the hospital digital dashboard, what types of data displayed?‫وﺟود اﻟﺷﺎﺷﺎت ﺗدل ع ﺗﻘﯾم اﻻداء‬
A. Staffing
B. Financial
C. Performance
D. Knowledge-based

// 2996699650 ‫ ﻣﺣﻣد ﺳﺎﻣﻲ‬/‫أ‬

14) In the Emergency Unit a nurse made an error that lead to an admission order for the client
to be on a venous 0 unit thromboembolic protocol is not processed. Two days after, a nurse
notices the omitted order for heparin 50s subcutaneous every eight hours which of the
following statement best describes the appropriate follow-up?]‫ﻧﺎﺳﯾن ﻧﻌطﯾﮫ اﻟدواء ﻟﻣدة ﯾوﻣﯾن وش اﺳوي‬
‫ اﻋﻠم اﻟﺳوﺑر‬,
A. " I will contact the supervisor immediately about this error``
B. " I need to contact a physician and complete a variance report``
C. ✅ I am too busy to complete a variance report. I'll do it next " ✅ eek✅
D. ✅ I am so glad I didn't make that mistake, that other nurse Is going to be in ✅ ✅ ✅ ✅ ble✅

15) A nurse manager assigns tasks according to clinical compet nurses which of the following is
the management function?

A. Delegating
B. Evaluating
C. Planning
D. Controlling

16. What must be known on legal points of delegation when process to a new nurse?
A. Evaluation of performance of delegate by clients
B. Actual time it takes to complete the task by delegat
C. Institution definition of the job description of
D. Number of times that the delegate has previously task

17. A nurse manager includes staff in decision-making process what unit structure is used?
A. Divisional
B. Centralized
C. Functional
D. Decentralized

18) The nursing director wants to evaluate the quality of nursing care at the in-patient areas.
The management team will evaluate on quarterly basis the documentation and the
relationship between the patient's length of stay and the quality of care which of the
following is the most important data source to identify the quality of care?‫ﻋﺷﺎن اﺷوف ﻧﯾﺟﺔ ﺟودة‬
‫اﻟﺧدﻣﮫ‬
A. Patient's complaints and time taken to resolve them
B. Patient's satisfaction level at the time of discharge‫ﻣدى رﺿﺎء اﻟﻣرﯾض‬
C. Details of nursing notes for patient's progress
D. Relation between incidents and cost of care

19) Which type of study design provides the strongest evidence?


A. Qualitative study
B. Randomized control trial
C. Systematic review of descriptive studies D. Systematic review of correlational studies

// 2996699650 ‫ ﻣﺣﻣد ﺳﺎﻣﻲ‬/‫أ‬

20. The head nurse of a Coronary Care Unit delegated the staff a senior nurse in that unit what
initial step must the head nurse implement before?
A. Check the hospital policies for delegating tasks
B. Explain the task to the senior nurse
C. Negotiate with the senior nurse
D. Take the signature of the senior nurse

21. Which of the following is the most appropriate action for a head nurse starting at a new
hospital?‫وش ﻧﺳﻲ اول ﺧطوه اذا ﺟﺎت ھد ع ﻣﺳﺗﺷﻔﻰ ﺟدﯾد‬
A. Make immediate change at the unit
B. Plan and coordinate new strategies
C. Assess unit activities for at least three months
D. Ask about the previous head nurse managerial style

22. a nurse is completing the preoperative checklist for one of the patients who ring is wearing a
ring. What is the most appropriate action?
A. Give ring to security office
B. Lock ring with patient's valuables
C. Call patient's family to give them the ring
D. Respect patient's choice and leave ring on patient's finger

23. The head nurse meets with staff nurses to discuss ways to improve communication among
shifts which of the following statement best exemplifies the final stage of conflict
management?
A. "We need to clearly define the nature of the conflict
B. I will evaluate the outcomes of the strategies used monthly
C. Let us create a time line for the implementation of our strategies
D. I have to force you to follow the rules to resolve the issue.

24) A nurse manager assigns tasks according to clinical competencies Of the nurse. Which of
the following is the management function? ‫اﻟﺗﻔوﯾض‬A. Delegating
B. Evaluating
C. Planning D. Controlling

25) A nurse manger is preparing and writing a plan for dealing disaster (code red). Which
procedure is the top priority for the nurse manager the plan?‫ﻟﻣن ﯾﻧﺣرق اﻟﻣﺳﺗﻔﻰ ﻣﯾن ﻻزم ﯾﻛون‬
‫ﻋﻧده ﻋﻠم ﺑﮭذا اﻟﻛود‬
A. Infection control
B. Staff orientation
C. Patient education
D. Patient relocation

// 2996699650 ‫ ﻣﺣﻣد ﺳﺎﻣﻲ‬/‫أ‬

26) nurse manager was not happy about low results of ✅ a✅ ien survey. Which of the
following is the first step for nurse manager?‫ﺗﻘﯾم اﻟﺑﺷﯾﻧت ﻏﯾر ﺟد‬
A. Schedule meeting with staff
B. Start changes to improve
C. Review evaluation details‫اﻓﮭم ﻟﯾش ﻏﯾر ﺟﯾد‬
E. Report to director

27. A nurse wants to delegate a post op patient from major surgery to assistant

nurse, what task she will delegate?‫ﻟﻣن اﺟﻲ اﻓوﺿﮫ اﻓوﺿﮫ اﻋطﯾﮫ اﻗل وظﯾﻔﮫ‬

A- ambulate the patient ✅ ✅


B-obtain vital signs.
C- check patency of NGT

28. A nurse accidentally dropped a medication ampule, informed the charge nurse report form.
Which of the following best describes the nurse role?

A. Responsiveness
B. Timely decision making
C. Profession accountability
D. Abiding by moral obligation

29. Group of health care givers provide care for group of patients have same

diagnosis at same setting. What is this considering?‫ﻋﻧدي ﻣﺟﻣوﻋﮫ‬

A. Team B. Case management C. Primary D. Functional Answer: B

30. As a manager, she focuses her energy on both the quality of services

rendered to the patients as well as the welfare of the staff of her unit. Which of the following
management styles does she adopt?‫ﻣﯾن رﻛز ع ﺟودة اﻟﺧدﻣﮫ واﻟﻧﺎس‬

A. Country club management


B. Organization man management
C. Team management
D. Authority-obedience management
Answer: C

// 2996699650 ‫ ﻣﺣﻣد ﺳﺎﻣﻲ‬/‫أ‬

31.The head nurse of a Coronary Care Unit delegated the staff a senior nurse in
That unit What initial step must the head nurse implement before?‫اﻟﮭد ﺑﯾﻔوض اﻻﺳف‬
A. Check the hospital policies for delegating tasks.
B. Explain the task to the senior nurse.
C. Negotiate with the senior nurse.
D. Take the signature of the senior nurse.

32. Female patient will do the breast implant surgery and she tell the nurse not to
tell her family about the surgery. What is the nature of this action?
A-confidentiality
B-ethical dilemma
C-illegal nursing practice
D-medico-legal practice
Answer: A

33. Nurse supervising on assistant nurses and delegate assignment to him and
say ―diving up patient's assignment." what type of leadership‫ﯾﻌﻣﻠو ع ﻛﯾﻔﮭم ﻣن دون م‬
‫ﯾﺗﺑﻌوا أي ﺗﻌﻠﯾﻣﺎت‬
A. Autocratic
B. laissez-faire
C. Situational
D.transformational2

34. A nurse manager in medical unit is not satisfied with the way things in her unit.
Patient satisfaction rate seductive months and staff morale is at its lowest. He
decides to plan and initiate changes that will push for a turnaround in the condition of the
unit. Which of the following actions is a priority for Henry?‫م ﻓﯾﮫ رﺿﻰ ﻣن اﻟﺑﯾﺷﻧت زﻋﻼن وش اﺳوي‬
‫ﻻزم اﺷوف اﻟﺗﻔﺎﺻﯾل ﺑﻌدﯾن اﺳوي ﻣﯾﺗﻧق ﻟﻼﺳﺗف‬
A. Call for a staff meeting and take this up in the agenda.
B. Seek help from her manager.
C. Develop a strategic action on how to deal with these concerns.
D. Ignore the issues since these will be resolved naturally

35. What is the Role of head nurse?


A. Ambulate the patient
B. Monitor ecg for the patient
C.Recruiting the nurses
D. Allocate the nurse to the patient‫ﺗوزع اﻟﻣﮭﺎم ﻛل ﻣﻣرض ﻟﻣرﯾض‬

36. Stephanie is a new Staff Educator of a private tertiary hospital. She conducts
orientation among new staff nurses in her department. Joseph, one of thenew staff
nurses, wantsto understand the channel of communication, span of control and lines of
communication. Which of the following will provide this information?‫ﻣﻣرﺿﺎت ﺟدد وﺻف‬
‫وظﯾﻔﻲ‬
A. Organizational structure
B. Policy

// 2996699650 ‫ ﻣﺣﻣد ﺳﺎﻣﻲ‬/‫أ‬

C. Job description
D. Manual of procedures

37. The head nurse is too busy to give evaluation to her staff. She gave to everyone
average evaluation. What is the evaluation type that the head nurse used?‫اﻟﮭد اﻋطت‬
‫اﻟﻛل ﻧﻔس اﻟﺗﻘﯾم ﻻﻧﮭﺎ ﻣﺷﻐوﻟﮫ‬
A. Halo effect‫ﻟﻣن اﻋطﻲ ﺗﻘﯾم اﺟﺎﺑﻲ ﻋﺷﺎن ﺷﻛﻠﮫ ﺣﻠو‬
B. Horn effect‫ﻏﯾر اﺟﺎﺑﻲ‬
C. Central tendency ‫ﻋﺷﺎن ﻛﻠﻣﺔ ﻣﻌدل اﻓرﯾﺞ‬

38-A nurse who has been working in the unit for 15 years has been receiving a lot of
complaints from patient and staff. She‘s meeting with the nurse manger. Which
statement the nurse manger would start with the appraisal?‫ ﺳﻧﮫ وف‬15 ‫ﻓﯾﮫ ﻣوظﮫ ﻟﮭﺎ‬
‫اﻟﺳﻧوات اﻻﺧﯾره اﻟﻣوظﯾﻔﯾن ﯾﺷﺗﻛون ﻣﻧﮭﺎ وﻗﺎﺑﻠﺗﮭﺎ وش اﺳﺋﻠﮭﺎ‬
A. ―How do you view your nursing care?ǁ ✅ ‫وش ﻣدى ﻗﺑوﻟك ﻟﻠﺧدﻣﮫ اﻟﻠﻲ ﺗﻘدﻣﯾﮭﺎ‬
B. ―You don‘t deserve to be in this unitǁ
C. ―I‘ve received a lot of good feedback back about the way you administer medication

39. During home visits nurse suspects that mother and child are being abused by
husband. Which question should the nurse ask?‫ﺗﺗﻌﻧف ھﯾﺎ وطﻔﺎﻟﮭﺎ وش اﺳﺋﻠﮭﺎ‬
A. ―Is there anything else bothering you you want to tell me about?ǁ
B. ―Are you being abused by husband?ǁ
C. ―Does your husband argue with you beat you and your son?ǁ ✅ ‫اﻗوﻟﮭﺎ اﻧﺗﻲ ﺗﺗﻌﻧﻔﻲ‬
D. ―Is your husband abusive?ǁ

40. Which of following is Directing function ?


-Lead and evaluate subordinates ✅ ‫ﻣن اﻋﻠﻰ ﻣﻧﺻب ﻟﻼﻗل‬

41. What key procedure for Occupational‫ ﻟﺣﻣﺎﯾﺔ‬health nursing to protect health and
security for working population?
A.Health promotion and prevention B.Health hazard identification
✅ C.Cost containment , job productivity D.Eliminate complaints

42. During Cardiopulmonary Resuscitation (CPR) for a 75-year-old man in the


Emergency Department, the doctor introduced himself as a leader for the CPR what is
the most appropriate leadership style for this situation?
A. Autocratic
B. Democratic
C. Bureaucratic
D. Laissez-faire

‫أ‪ /‬ﻣﺣﻣد ﺳﺎﻣﻲ ‪// 2996699650‬‬


Psychiatric
1. The psychiatric patient is receiving Benzodiazepine drug. The nurse should instruct patient
that drug affected by which of the following?‫اش ﯾﻘﻠل ﻣن ﻣﻔﻌول اﻟدواء‬
A. Alcohol ✅
B. Tobacco
C. Nicotine
D. Caffiene

2.Patient admitted to inpatient ward diagnosed with schizophrenia. What is the first intervention
for patient?‫اﻟدوﺑﺎﻣﯾن ﯾﻛون ھﺎي‬
A. Observe patient behavior ✅
B. Give health education about medication

3.Patient with Alzheimer's disease. What is the cause that affect brain function (the defect)?
A. Absent of nerve cells
B. Destruction of neuronal ✅ ‫ﺗﺣطم ف اﻟﺧﻼﯾﺎ اﻟﻌﺻﺑﯾﮫ‬

4. Which of the following Most of the time sign associated with Alzheimer's disease?‫اش اﻟﻠﻲ ﯾرﺗﺑط‬
‫ﺑﺎﻟزھﺎﯾﻣر‬
A. Depressed mood ✅
B. Act out of behavior

5. A 26 years old married woman is admitted in the plastic surgery for the correction of burn
strictures and skin g rafting on her neck and face under general anesthesia. While discussing
the treatment with her, the plastic surgeon explained that she will have a series of surgeries
but she needs to be on family planning until the treatment is completed. the patient asked the
nurse whether she will be normal again. What initial assessment is required?‫ﻣﺗﻰ ارﺟﻊ ﻟوﺿﻌﮭﺎﻟطﺑﯾﻌﮫ‬
‫ﻣﻌﻧﺎﺗﮫ ﻋﻧده ﻗﻠق وش اﺳوي‬
Need for psychological support to reduce anxiety ✅

6. A 69-year-old patient was discharged from instructions to have follow-up physiotherapy sessi
paralysis as a result of the brain stroke. A few physiotherapy sessions, he showed fast
rsecovery sadly that despite his progress he will remain which of the following should?the
prioritize patient
A. Support for health maintenance
B. Increase activity and exercise
C. Identify coping mechanism
D. Enhance self esteem

7. A 27 years old female brought to the Emergency Rooaccompanied by her husband. He


described that she had marked Weight loss with episodes of emesis in the past three
months. She is diagnosed as having anorexia. She reported feeling Febrile, but had not
measured her

// 2996699650 ‫ ﻣﺣﻣد ﺳﺎﻣﻲ‬/‫أ‬

temperature. Her White Blood Count was 11,000/mm3. Which of the following most Likely
describe her diagnostic criteria for her anorexia?‫ﯾﺣط ﯾده ف ﻓﻣﮫ ﻋﺷﺎن ﯾﺳﻔرغ‬
A-Restricting food intake
B-Fear of gaining weight
C-Problems with body image
D-Binge eating disorder

8. A 26 year-old married woman is admitted in the plastic surgery for the correction of burn
strictures and skin grafting on her neck face under general anesthesia. While discussing the
team, the plastic surgeon explained that she will have a surgeries but she needs to be on
family planning until the treat the patient asked the nurse whether she will be normal again
what initial assessment is required.
A. Detailed history and physical examination
B.patient‘s acceptance for the treatment
plan
C. Need for psychological support to reduce anxiety
D. Family‘s involvement and consent for her treatment

9) A high school girl, who has fears of being obese thin, visited the primary healthcare center
with her the mother reports that her daughter refuses to with the family, and often pretends
being a sleep Which of the following disorders best describe girl?‫ﻧﺣﯾف وﺗﺗﻘﯾﻎ وش ﯾﻌﻧﻲ‬
A. Bulimia
B. Obesity
C. Substance Abuse
D. Anorexia Nervosa

10) A 28-year-old women is accompanied by her mother to the ward. She is admitted for her
affective disorders with anger recent events and stammering spells. She has panic followed
by apathy and listless Ness. How should the nurse manage the patient?‫؟ﻏﺎﺿب اﺳﺗﺧدم‬

A. Be supportive and use therapeutic communication


B. Improvise sign language to control forgetfulness
C. Maintain calm environment and avoid argument
D. Anger management and speech therapy

11. A high school girl, who has fears of being obese and abased with being thin, visited the
primary healthcare center with her mother for counselling The mother reports that her
daughter refuses to eat during meal times with the family, and often pretends being a sleep to
skin meals. Which of the following disorders best describe girl condition?
A. Bulimia
B. Obesity
C. Substance Abuse
D. Anorexia Nervosa

// 2996699650 ‫ ﻣﺣﻣد ﺳﺎﻣﻲ‬/‫أ‬

12. A 28-year-old man and his wife involved in which his wife was killed. The client is being
Care Unit for multiple rib fractures and a broken which room his wife is located. Which of
the following is the most appropriate?‫ﻣﺎﺗت زوﺟﺗﮫ اﻟدﻛﺗور ﯾﺑﻠﻐﮫ‬
A. Your wife is not in the hospital
B. I'm sorry, but your wife did not survive
C. "I need to get your family so that you can wife
D. The doctor will be talking to you about located✅

13. Patient with Manic bipolar disorder. He is hyperactive, too much moving, talk

A lot and he has loss of appetite, very thin . The patient also don't sleep. What is the most
appropriate nursing diagnosis?

A. High risk for suicide


B. High risk for injury✅
C. Nutritional disturbance
D. Alter sleep pattern

14. patient diagnosed with delirium sees the intravenous (IV) tubing and believes it

to be a snake. How should the healthcare provider document this behavior?‫ﺗﺗﺧﯾل‬

‫اﻟﺳﻣﺎﻋﮫ ﻛﺎﻧﮭﺎ ﺛﻌﺑﺎن‬

A. Hallucination B. Illusion ✅ C. Confusion D. Delusion

15. Nurse notes some students with anorexia nervosa what does she do??

A. Push them to eat.


B. One to one superior during eating
C. Let students eat with them the same meals.

16- aschizophrenic patients saysi feel like my arms and limbs are detached from

my body and i don't feel like i am a alive and it made me avoid socializing" the

nurse interpreted this as:‫م ﯾﺣس ﺑﯾدﯾﻧﮫ وروﺟﻠﮫ وم ﯾﺣس اﻧو ﻋﺎﯾش‬

A- Depersonalization‫ﺗﺑدد اﻟﺷﺧﺻﯾﮫ‬
B- B-Loss of association
C- Dissociative identitydisorder
D- Dissociative amnesia

// 2996699650 ‫ ﻣﺣﻣد ﺳﺎﻣﻲ‬/‫أ‬

17. Patient with somatization‫ اﻧﺗﻘﺎﺋﻲ ﺟدا‬.Whatis the best action?


A. Cognitive development ‫ﺗطوﯾر ادراﻛﻲ ﺣﻔظ‬

18.Psychiatric patient doesn‘t like her nose and mouth shape. She has false belief
of disturbed body image and counselled many doctors' for doing operation. Doctor's
not accepted her due to no need everything is normal. What is type of disorder?
A— Pain B—
Conversion
C— Body dysmorphic disease
D — Hypochondriasis

19-Patient with Dissociative disorder. Which of the following nursing diagnosis should
the nurse expect?‫ﻣوﻣﺗﻘﺑل ﺷﻛﻠﮫ‬
A. Impaired cognitive process
B. Self image deficit ✅

position
1 Most recommended position for a client with severe ascites? ‫ﺗﺟﻣﻊ ﺳﺎﺋل ف اﻟﺑطن‬
A Side lying. B.Sim‘s.
C.Dorsal.
D.Fowler‘s.

2. What is COPD position?


A- Semi fowler
3. B- Supine
C- Prone
D- High fowler✅

4. A 65-year-old man is undergoing pre-operative preparation for endoscope procedure in which


the physician will visualize the lar and distal part of the small bowel with a camera attached to
the flexible tube Which of the following positions is the most appropriate?
A. Left lateral Sim's
B. Right lateral recumbent
C. Trendelenbreg
D. Prone

5. A nurse is preparing to insert a nasogastric tube into a client. The nurse places the client in
which
Position for insertion:
A-high Fowler's
B-low Fowler's
C-right side
D-supine with the head flat

// 2996699650 ‫ ﻣﺣﻣد ﺳﺎﻣﻲ‬/‫أ‬

6. Position for lumbar puncture?!

A. lateral recumbent position

B. prone position

-supineposition

7. What is the Position during thyroidectomy??‫اﺛﻧﺎء اﻟﻌﻣﻠﯾﮫ‬

A. Supine with hyper flexion of neck‫ﺑﻌد اﻟﻌﻣﻠﯾﮫ‬

B. Supine with hyperextended neck ‫اﺛﻧﺎء اﻟﻌﻣﻠﯾﮫ‬

C. Semi fowler

D. Lateral with slightly flexion neck

8. What is the position for adenoidectomy??

A. Semi Fowler B. Sitting C. prone‫ع اﻟﺑطن‬

Nutrition
1. Which of the following food rich in potassium?
A. Orange ✅
B. Tofu
C. Butter
D. Milk

2. A nurse is assigned to care for a patient who Nutrition (TPN).


The patient was cyanosed. Which of the following conditions is
a TPN complications?‫ﻋدﻟﻧﺎه ﻓوق‬
A. Hyperglycemia
B. Pneumothorax
C. Hypervolemia
D. Infection

3. The nurse is caring woman that has cancer, and she is under Chemotherapy.

// 2996699650 ‫ ﻣﺣﻣد ﺳﺎﻣﻲ‬/‫أ‬

She is complaining anorexia and the patient has low weight. What should the nurse

instruct her???

A. Eat small meals every day.


B. Eat large meals every day.
C. Eat if you are hungry.
D. Eat your favorite food

4. The nurse is giving instruction for a pregnant woman. Which of the following

should be avoided during pregnancy?

A-raw uncooked met.


B-clean well fruit and vegetables
C-cooked fish and chicken

5. What is the Position during thyroidectomy??

A. Supine with hyper flexion of neck


B. Supine with hyperextended neck
C. Semi fowler
D. Lateral with slightly flexion neck.

6. What is chest assessment for patient with Vitamin D deficiency?

A. Dull sound B. Rosary ✅ C. Crackle D. Chest deformity

7. year-old child was admitted with Nephrotic Syndrome. A nurse noticed that the

child has slight facial puffiness with mild pitting edema on his hands and feet.

there was no distended abdomen diet the nurse should order for the child?

A. High protein, high salt diet


B. Low protein, low fiber diet
C. Low protein, normal salt diet
D. Normal protein, low salt diet Answer: D

myringotomy :Needle for aspiration of fluid from ear Procedure that makes an incision into the
tympanic membrane usually done to relieve inflammation in the middle ear
‫اﻻذن اﻟوﺳطﻰ ﻓﯾﮭﺎ اﻟﻠﺗﮭﺎب ﻛون ﺳواﺋل ﻧﺳﺣﺑﮫ ﺑﺎﺑره‬

Myringoplasty : surgical repair of the eardrum with a tissue graft. this procedure is performedto
correct hearing loss. It is also called tympanoplasty.‫ﯾﺣطون ف طﺑﻠت اﻻذن زي اﻟﻘراف ﻟﻠﻲ ﻋﻧدھم ﻣﺷﻛﻠﮫ‬
‫ﺑﺎﻟﺳﻣﻊ‬

// 2996699650 ‫ ﻣﺣﻣد ﺳﺎﻣﻲ‬/‫أ‬

advocate nurse: defined as protecting by expressing and defending the cause of another.
Representing the client's needs and wishes to other healthcare professionals. Helping clients
exercise their rights‫ﻣﺗﻰ ﺗﺣﻣﯾﮫ ﺗﺣﻣﯾﮫ ﻣن اﻟﺑﯾﺋﮫ اﻟﻣﺣﯾطﮫ‬

hemiparesis? A.Weakness on one side of the body‫ ✅ ﺣﻔظ‬B.Pralysis on one side of the body

// 2996699650 ‫ ﻣﺣﻣد ﺳﺎﻣﻲ‬/‫أ‬

You might also like